TExES: Special Education EC-12 Practice Test 1 (Flash Cards)

Ace your homework & exams now with Quizwiz!

At what age does the Texas state law say transition planning should begin?

A. 14 B. 15 C. 16 D. 18 (A) The correct response is (A). In 2011, the Texas legislature made 14 the required age for transition planning to begin.

Given the algebraic equation 3x + 4x - 1 = 34, the value of x is

A. 3 B. 4 C. 5 D. 10 (C) The correct response is (C). To solve for x in (3x + 4x - 1 = 34), the first step would be to subtract 1 from the left side of the equation and add it to the right side of the equation, giving us (3x + 4x = 35). We then add 3x and 4x on the left side of the equation to get (7x = 35). We then divide both sides by 7 to get (x = 5). (A), (B), and (D) are incorrect because they are not the value of x in this equation.

Which of the following areas is not a type of assessment accommodation?

A. Presentation B. Scheduling C. Content D. Timing (C) The correct response is (C). Content will never be a type of accommodation because accommodations never alter content. (A), (B), and (D) are all ways that accommodations can change the way a test is given.

Use the information below to answer questions 115-117. Mrs. Miller, a fourth-grade teacher, asks her students to conduct a survey of heights for a sample of fourth-grade boys and girls. Mario surveys 18 students: 9 boys and 9 girls. The following chart shows his results. Boys Girls 46 47 49 48 50 49 50 51 51 52 52 54 53 54 54 56 56 58 Based on the chart, the median height for the boys is

A. 50 inches.B. 51 inches.C. 52 inches.D. 53 inches. (B) The correct response is (B). A median number is the middle number in a sorted list of numbers. There are nine boys and the fifth number, directly in the middle, is 51. This is the median height. (A), (C), and (D) are incorrect as they are not the middle number in the sorted list from shortest to tallest boy.

Use the information below to answer questions 115-117. Mrs. Miller, a fourth-grade teacher, asks her students to conduct a survey of heights for a sample of fourth-grade boys and girls. Mario surveys 18 students: 9 boys and 9 girls. The following chart shows his results. Boys Girls 46 47 49 48 50 49 50 51 51 52 52 54 53 54 54 56 56 58 Based on the chart, the range of heights for the girls is

A. 9 inches. B. 10 inches. C. 11 inches. D. 47 inches. (C) The correct response is (C). The range is calculated by finding the difference between the largest number in a data set and the smallest number. For the girls, the largest number is 58 and the smallest number is 47, so if you subtract 47 from 58 (58 - 47 = 11), the range is 11 inches. (A), (B), and (D) are incorrect because they are not the difference between 58 and 47. Choosing (D) indicates a conceptual misunderstanding of range since 47 is the smallest number in the data set.

According to the National Reading Panel's 2000 report, "Teaching Children to Read," the evidence to support phonemic-awareness instruction is statistically significant. Which of the following strategies represents phonemic awareness instruction?

A. A teacher circles all the spelling errors students make in their first drafts. B. A teacher says several rhyming words aloud and asks students what they notice about the words. She then says the word "wet" and asks students to call out words that rhyme with "wet." C. A teacher sits beside a student who is reading out loud and models any word the student stumbles over. D. A teacher reads a book and then asks students to point out their favorite parts. (B) The correct response is (B). Phonemic awareness is the ability to hear and recognize phonemes. Recognizing rhyming sounds involves recognizing the phoneme that is making the rhyme. Calling out words that rhyme with a target word demonstrates an understanding of a phoneme. (A) is incorrect because circling spelling errors would be a part of the editing phase in the writing process. (C) is incorrect because sitting with a student who is reading and helping with unknown words means the student has progressed far beyond phonemic awareness and is now decoding. (D) is incorrect because it describes a postreading activity that allows students the opportunity to react to the book the teacher just shared.

Research on effective math instructional strategies has indicated that explicit instruction is critical. Which of the following lessons is an example of an explicit teaching strategy?

A. A teacher has students complete worksheets with operational drills for 20 minutes every day.B. A teacher gives students 20 colored squares and then steps back and observes what the students do with the squares.C. A teacher demonstrates how to solve an addition problem using a number line, talking out loud through each step of the addition process.D. A teacher assigns students homework over the weekend, telling them to find examples of math in the world around them. (C) The correct response is (C). Explicit instruction involves helping students see how a math strategy is applied step-by-step. Modeling and verbalizing each step gives students a concrete picture of how to solve a problem. (A) is incorrect because giving students independent work is not explicitly showing them how to complete problems. (B) is incorrect because stepping back and observing what students do with manipulatives is an implicit approach. Students are expected to come up with what the problem and solution are on their own. (D) is incorrect because asking students to see how math works in the everyday world, while motivating, is not a step-by-step, explicit strategy. This "making connections" exercise is largely implicit.

Lisa, an eighth grader recently diagnosed with ADHD, often exhibits inappropriate attention-seeking behaviors in her classes. In order to help create an IEP, Lisa's special education teacher wishes to know more about why Lisa engages in these behaviors and what strengths Lisa has that could be leveraged to help her behave more appropriately. What approach to assessment is the teacher likely to use?

A. Alternative B. Summative C. Functional behavioral D. Criterion-referenced (C) The correct answer is (C) because functional behavioral assessment seeks to determine what function particular behaviors serve, and to understand any strengths and weaknesses relevant to maintaining or changing those behaviors. Answer choices (A), (B), and (D) might provide helpful information and could be part of a functional behavioral assessment, but, broadly speaking, the latter is what is needed in this situation.

A teacher in an autism unit has several students who become frustrated whenever their routines change. Two of them are nonverbal. What is one technique the teacher can use to help her students become more flexible?

A. Applied Behavior Analysis B. Floortime C. Circle time with calendar and weather D. Visual schedule (D) The correct response is (D). A visual schedule is a research-based technique that helps students with autism understand the plan for the day and what activities have to be completed before others can occur. This is especially helpful when a daily routine changes and a teacher wants to give forewarning. Reviewing the schedule and then removing the card representing each major activity after it is completed allows students to stay on task and have a sense of control over what is happening. (A) is not the correct answer because the purpose of Applied Behavior Analysis is skill acquisition and, by its nature, it relies on routines. It does not teach flexibility. (B) is not the correct answer because the purpose of floortime instruction is to get on the same level as children with autism and engage them. The focus is not on building flexibility but, instead, on building human interaction and social skills. (C) is not the correct answer because circle time with calendar and weather is just one routine activity on a schedule. By itself, circle time does not teach flexibility.

ARD stands for

A. Assessment, review, and determination B. Admission, review, and dismissal C. Admission, reevaluation, and documentation D. Assessment, review, and documentation (B) The correct answer is (B). The other options are not standard phrases.

How often must a student's transition plan be updated?

A. At the beginning of each semester B. Once a year as part of the ARD C. At ages 16, 18, and 21 D. Whenever the student gets a new job experience (B) The correct response is (B). IDEA 2004 requires that a student's transition plan be updated annually along with the student's IEP. This is part of the regular ARD process in Texas. (A), (C), and (D) are incorrect choices since they do not account for the annual update requirement.

What ages does an Individual Family Service Plan apply to?

A. Birth to 3 B. 4-5 C. 14-18 D. 21 and older (A) The correct response is (A). An Individual Family Service Plan is provided for children birth to age 3 as part of early childhood services for children with developmental delays. At age 3, the IFSP is replaced by an IEP. (B), (C), and (D) are incorrect as they all represent ages covered by an IEP.

Which of the following skills would be an example of a child's gross motor capabilities?

A. Drawing with a crayon B. Skipping along a sidewalk C. Talking to a friend D. Solving an addition problem (B) The correct response is (B). Gross motor skills involve large body movements. Skipping is a gross motor ability. (A) is incorrect because using one's fingers to hold a crayon and draw involves fine motor skills. (C) and (D) are incorrect because they have nothing to do with large body movements.

A third-grade student with learning disabilities turns in the following piece of writing after 40 minutes of classroom writing time:Based on the student's writing sample, at what stage of spelling would the special education teacher most likely place this student?

A. Emergent B. Letter Name-Alphabetic C. Within-Word Pattern D. Syllables and Affixes (B) The correct response is (B). Based on the writing sample, the student is in the latter stages of the letter name-alphabetic stage of spelling. The student is generally starting the words with the correct sounding letter and even has some of the inner consonants and vowels correct. However, there are many interior letters missing, and all of the silent "e's" are missing. (A) is incorrect because the student has clearly progressed past the emergent spelling stage where letters do not necessarily correspond to sounds. (C) and (D) are incorrect because the writer has not moved to these higher levels of spelling yet.

Parents of a child with math learning disabilities want tips on how to help their child with homework. Which of the following special education journals would have articles that would be most appropriate to share with these parents?

A. Exceptional Children from CEC B. Educational Researcher from AERA C. TEACHING Exceptional Children from CEC D. The Examiner newsletter from IDA (C) The correct response is (C). TEACHING Exceptional Children from the Council for Exceptional Children is a practical journal that features research-to-practice information and materials for classroom use. Parents will find articles fairly easy to navigate, and several articles are specific to math strategies. (A) is incorrect because Exceptional Children is a scholarly research journal. Parents may find the articles difficult to navigate, and the strategies discussed may be difficult to translate into practice. (B) is incorrect because Educational Researcher is the American Educational Research Association's scholarly research journal. Articles are difficult for laypeople to understand and cover topics broader than special needs. (D) is incorrect because The Examiner is a newsletter published by the International Dyslexia Association, and it focuses on reading issues, not math issues.

By definition, instruction at a slightly higher level than a student's current skill/knowledge level creates which of the following?

A. Frustration B. Interest C. Cognitive demand D. Rapid learning (C) By definition (C) is correct. Options (A), (B), and (D) are incorrect because such instruction does not necessarily frustrate students, interest them in the material, or result in rapid learning, although such outcomes may sometimes occur.

Which of the following is an example of a writing-assistive technology? (Select all that apply.)

A. Graphic organizing software B. Alternative keyboard C. Speech-to-text software D. Recorded book (A,B,C) The tools described in (A), (B), and (C) can each function well as a writing-assistive technology. (D) is incorrect because recorded books will primarily support reading rather than writing.

A fourth-grade teacher assigns several three-digit multiplication problems for her students to complete. In reviewing the papers, the teacher finds one with over 50 percent of the answers incorrect. Attempting to see what caused the mistakes, the teacher notices that the student has misaligned many of the numbers. The computation that was done is correct, but the wrong numbers were often multiplied or added together. In reflecting on this student's past math performance, the teacher recalls that geometry also proved difficult for the student. The assumption that would most likely be true about this student is which of the following?

A. Hates math and does not put forth effort to do the work neatly B. Seems to be exhibiting visual-spatial deficits that are affecting academic performance C. Has Attention Deficit Hyperactivity Disorder D. Requires handwriting supports (B) The correct response is (B). Visual-spatial deficits cause difficulty with conceptualizing geometric figures and aligning numbers. Based on the teacher's observations of this student, visual-spatial deficits seem likely. (A) is incorrect because the student is clearly trying to do the work, despite frustrations. (C) is incorrect because the nature of the errors points to cognitive deficits, not to lack of attention. (D) is incorrect because the misalignment is not due to poor fine motor skills but to lack of visual/spatial clarity.

Which of the following lists reflects an ordering from least to most restrictive environments?

A. Home instruction, separate facility, resource room, small group setting, regular education classroom B. Small group setting, regular education classroom, separate facility, home instruction, resource room C. Regular education classroom, small group setting, resource room, separate facility, home instruction D. Resource room, home instruction, small group setting, regular education classroom, separate facility (C) The correct ordering is given in (C). Options (A), (B), and (D) do not begin with the least restrictive environment, nor is their ordering correct.

What kinds of knowledge might be acquired through transition services? (Select all that apply.)

A. How to use public transportation B. How to manage money C. How to maintain friendships D. How to behave on the job (A,B,C,D) All four options are correct, as each one may be necessary for a student's independent living.

Mr. Garcia has his fourth-grade students read to themselves the following sentence from a Texas history text: "Juneteenth is the oldest known celebration commemorating the ending of slavery in the United States." Mr. Garcia then asks the students what the term "commemorating" means in the sentence. He suggests they look at other words in the sentence to help them find the meaning. What reading comprehension strategy is Mr. Garcia teaching his students to use?

A. Identifying the main idea B. Using context clues C. Visualizing D. Blending (B) The correct response is (B). Context clues are the words around an unknown word in a text. By having students find surrounding words that can help them determine the likely meaning of "commemorating," Mr. Garcia is building students' independent reading strategies. (A) is incorrect because the main idea is not related to the meaning of a word in a passage. It is the central message of the entire piece. (C) is incorrect because visualizing is a strategy used during reading to help the reader picture what is happening in the text. (D) is incorrect because blending is an aspect of phonics instruction in which letter sounds are blended together so that students can decode words.

What kinds of information can transition assessment provide? (Select all that apply.)

A. Information about strengths and weaknesses B. Information relevant to scaffolding instruction C. Information about how to support independent living D. Information about demographic background (A,C) The correct responses are (A) and (C). Information about strengths and weaknesses, and more specifically about how to support independent living, will help focus transition planning. (B) is incorrect unless the student will be seeking further instruction following the transition to independent living. (D) is incorrect because this information will already be available.

Lee takes medicine for asthma that makes him drowsy and interferes with his ability to pay attention in class, particularly in the mornings. Under what category might IDEA require Lee to receive special education services?

A. LD B. Intellectual disabilities C. OHI D. Attention disorders (C) The correct response is (C) because OHI (Other Health Impairments) includes medical conditions that impair attention in educational settings. The conditions described in options (A), (B), and (D) do not fall under the heading of OHI.

When a school changes the placement for a student with disabilities, must the parents be notified?

A. Notification of the parents is optional. B. Notification of the parents is a legal requirement, but further explanations are optional. C. Notification of the parents is a legal requirement, as well as an explanation of why the change in placement was made. D. Notification of the parents is a legal requirement, as well as an explanation of why the change in placement was made, what alternatives were considered, and how various factors determined the change. (D) The correct answer is (D). IDEA requires that parents receive more information than what is described in options (A), (B), and (C).

What does OHI stand for?

A. Only Healthy Individuals B. Oppositional Hyperactivity Indicator C. Organized Heart Implant D. Other Health Impairment (D) The correct response is (D). OHI stands for Other Health Impairment. This category includes a number of chronic or acute health problems that can adversely affect a child's educational performance.

When writing a behavior intervention plan for a student whose behavior impedes learning, what kind of strategies does IDEA 2004 require the IEP team to consider?

A. Positive behavioral interventions and supports B. Behavior logs C. Negative reinforcers D. Parental strategies (A) The correct response is (A). IDEA 2004 specifically states that the IEP team should consider positive behavioral interventions and supports to address behaviors that impede learning. The Texas Education Agency fully supports this statement and urges schools to adopt proactive disciplinary approaches. (B) is incorrect because, while behavior logs may be one of the forms of documentation used to justify a behavior intervention plan, they are not specifically required by IDEA 2004. (C) is incorrect because negative reinforcers are discouraged based on the latest research on student behavior. (D) is incorrect because, while some parental strategies work well and the IEP team includes the parents and listens to the parents' viewpoint, the team develops strategies to deal with the child's behavior at school.

Use the information below to answer questions 132 and 133. Mr. Aquino, an eighth-grade reading teacher, has his students create summary notebooks. For each chapter they read, they complete a simple chart listing who the main character is, what the main character's objective is, what obstacles stand in the way of the objective, and how the character deals with the obstacle. To help students who are struggling, Mr. Aquino has the students label the four columns of their charts with the words, "Who," "Wanted," "But," and "So." This helps cue students to fill in the applicable information. Here is a sample page from one of the student's notebooks for chapter three of the book she is reading: Who Wanted But So Kat has a crush on Rick and wants him to like her Rick is busy with basketball and doesn't notice her Kat decides to go out for cheerleader tryouts Which stage of the reading process is Mr. Aquino's strategy designed for?

A. PrereadingB. During readingC. After readingD. Emergent reading (C) The correct response is (C). Summarization is a strategy used after reading to pinpoint the gist of the storyline in a narrative text. (A) is incorrect because you cannot summarize a story before you read it. (B) is incorrect because you can have a sense of the overall summary as you read, but you cannot summarize the entire piece until you have finished reading it and stepped away to reflect on the text. (D) is incorrect because emergent reading is not a stage of the reading process. It refers to the level of a very beginning reader.

Which federal law replaced the No Child Left Behind Act (NCLB) in 2015?

A. Public Law 99-457 B. Elementary and Secondary Education Act C. Public Law 108-446 D. Every Student Succeeds Act (ESSA) (D) The correct response is (D) since ESSA replaced NCLB as a reauthorization of the original Elementary and Secondary Education Act of 1965.

Which of the following is likely to be an underlying cause of bullying behavior?

A. Racism B. Lack of a sense of control C. Dislike of other people D. Overprotective parent (B) The correct response is (B). Exhibiting bullying behavior often stems from a deep feeling of insecurity. Rather than face his own fears, the bully makes others feel bad and exerts power to try to establish a sense of control. (A) is incorrect because, while racism may coexist with bullying behavior, it is not the root cause. (C) is incorrect because, deep down, the bully wants to be liked but doesn't believe it is possible. (D) is incorrect as the root cause is the bully's own feelings of not being in control, not an external person's behavior.

A fourth-grade student with learning disabilities is struggling with language arts. The student likes to compose stories, but her handwriting is extremely difficult to read and she makes numerous spelling errors. What assistive technology would you recommend the child be evaluated for?

A. Recorded books B. Alternative keyboard C. Word processor with a spell checker D. Electronic math worksheets (C) The correct response is (C). Based on the description, the student's primary difficulty with writing appears to be fine motor (poor handwriting) and lack of the alphabetic principle or difficulty with memory (poor spelling). While special education testing can give a much better picture of the causes, a simple accommodation, such as providing access to a computer or word processor (where the student can type her papers and use a spell checker to identify misspelled words and correct them), can be provided with or without a special education designation. (A) is not the correct answer because the challenges indicate an issue with the act of writing, not with reading. (B) is not the correct answer because an alternative keyboard is designed for students with physical disabilities or limited reading skills. This student does not appear to need an adapted keyboard. (D) is incorrect because the issue is with writing, not mathematics.

A test administrator, per a student's IEP, administers the test in a location with minimal distractions. What type of assessment accommodation would this be?

A. Response B. Presentation C. Setting D. Timing (C) The correct response is (C). Administering the test in a place with minimal distractions is an accommodation in the setting or location. (A) is incorrect because an accommodation in response would be a change in how the student indicates his/her answers. (B) is incorrect because an accommodation in presentation would be a change in how the test questions are spaced or in how the directions are presented. (D) is incorrect because an accommodation in timing would mean that a student is given more time to take a test or given more frequent breaks than other students receive.

A behavior intervention plan (BIP) for Jeffrey describes the intervention that will be used to help him learn better impulse control. The BIP provides a detailed description of the nature and timing of the intervention. Which of the following is missing from this BIP? (Select all that apply.)

A. Results of interviews with the parents about what they consider the most appropriate approach to intervention B. Copies of articles describing interventions that would be appropriate for Jeffrey C. Baseline information about Jeffrey's lapses in impulse control D. Details of how the impact of the intervention will be measured (C,D) The correct responses are (C) and (D). A BIP should include information about the baseline frequency, duration, and intensity of the behaviors that will be the focus of intervention. In addition, the BIP should include information about how the intervention will be measured. However, the information described in options (A) and (B) are unlikely to be part of a BIP.

Mr. Boychuk is a seventh-grade mathematics teacher. He is assigned a co-teacher because he has several students with IEPs in his classes. Mr. Boychuk provides step-by-step explanations for each mathematical concept he presents. The co-teacher volunteers to illustrate the explanations on the interactive whiteboard while Mr. Boychuk gives the lesson. After the lesson, the co-teacher pulls together small groups and provides manipulatives. She then asks the students to use the manipulatives to demonstrate the steps Mr. Boychuk explained. She encourages students to narrate the steps as they demonstrate them. What instructional strategy is the co-teacher using?

A. Simulation B. Game-based teaching C. Multisensory approach D. Visualization (C) The correct response is (C). By providing visual reinforcements and concrete examples, the co-teacher is providing multiple ways for students to grasp the instructional concepts. (A) is incorrect because a simulation would involve students playing roles in a mock real-world application of the mathematical concepts. (B) is incorrect because the students are not playing a game. (D) is incorrect because the students have not been asked to form a picture in their minds or imagine a situation.

Under the Individuals with Disabilities Education Act, which of the following is NOT considered a disability category?

A. Speech/language impairment B. Intellectual disability C. Specific learning disability D. Post-traumatic stress disorder (D) The correct response is (D). Under the Individuals with Disabilities Education Act, there are fourteen disability categories used by the states to determine whether students aged 3 to 21 are eligible for special education and related services. (A) Speech/language impairments, (B) intellectual disability, and (C) specific learning disability are three of those fourteen. Only (D) is not a disability category under IDEA.

A special education teacher wishes to know how much a particular group of students is benefiting from a reading intervention, so that she can modify the intervention to better support the students. What type of assessment should the teacher use?

A. Summative B. Formative C. Alternative D. Norm-referenced (B) The correct response is (B) because the results of formative assessment are used to modify instruction, interventional approaches, and other educational activities. (A) is incorrect because summative assessment is used at the end of an instructional unit or intervention to determine impact. (C) is incorrect because it's not clear from the scenario that an alternative to a conventional form of assessment would be needed. (D) is incorrect because what is of most interest to the teacher is not the students' performance relative to established norms, but rather changes in the students' performance.

Use the information below to answer questions 109 and 110.A third-grade student with learning disabilities turns in the following piece of writing after 40 minutes of classroom writing time:After analyzing the writing, the student's special education co-teacher notes strengths to encourage and weaknesses to target. Which of the following aspects of the student's writing would be considered a strength?

A. Syntax B. Capitalization conventions C. Writing fluency D. Punctuation conventions (D) The correct response is (D). One of the patterns evident in the writing sample is the writer's correct use of punctuation. The fact that every sentence is punctuated correctly is definitely a strength. (A) is incorrect because the syntax of the sample is very simple. Sentences are repetitive and do not have varied beginnings. (B) is not correct because there are seemingly random mistakes in capitalization throughout the sample. (C) is not correct because the fact that the student wrote only 27 words in 40 minutes is a sign of a writer in need of intervention.

Mrs. Miller, a fourth-grade teacher, asks her students to conduct a survey of heights for a sample of fourth-grade boys and girls. Mario surveys 18 students: 9 boys and 9 girls. The following chart shows his results. Boys Girls 46 47 49 48 50 49 50 51 51 52 52 54 53 54 54 56 56 58 Based on the chart, which of the following statements is true?

A. The average height of the girls in this sample is higher than the average height of the boys in this sample. B. The mode of the heights of the boys is higher than the mode of the heights of the girls. C. Fourth-grade girls are taller than fourth-grade boys. D. None of the girls in the sample were 4 foot 8 inches. (A) The correct response is (A). The average, or mean, is calculated by adding up all the numbers in a data set and then dividing by the amount of numbers you have. In this case, when we calculate the average heights of the girls and boys, we get 52.111 for the girls and 51.222 for the boys. Thus, the average height of the girls in the sample is higher than the average height of the boys in the sample. (B) is incorrect because mode is calculated by the most common number. For the boys, this is 50 and for the girls this is 54. (C) is incorrect because this is a sample of 18 fourth-grade boys and girls. It is not a large enough sample to be representative of all fourth graders. (D) is incorrect because 4 foot 8 inches is equivalent to 56 inches, and one of the girls in the sample was 56 inches tall.

In a case where the school district and the parents disagree about a student's placement in special education or about the services the school district will provide, either the district or the parents may call for a hearing before an impartial hearing officer. Which of the following statements regarding the hearing is true, according to special education laws?

A. The hearing officer must not be an employee of the school district. B. The hearing officer will listen only to the parents and a school district representative. No one else may attend the hearing. C. The hearing cannot be open to the public. D. The hearing officer can only offer an opinion on the situation. It is up to the parents and the school district to resolve their own differences after the hearing concludes. (A) The correct response is (A). An impartial hearing officer cannot be an employee of a school district or any public agency that is involved in the education or care of the child. (B) is incorrect because any party to the hearing has the right to be accompanied and advised by legal counsel and by individuals with special knowledge regarding the problems of children with disabilities. (C) is incorrect because parents have the right to open the hearing to the public. (D) is incorrect because the hearing officer will issue a binding decision at the conclusion of the hearing.

Which of the following strategies can promote reading comprehension among students with learning disabilities? (Select all that apply.)

A. Use of semantic webs B. Instruction in prereading C. Direct instruction in decoding D. Sight-word drills (A,B,C) The correct answers are (A), (B), and (C). (D) is incorrect because sight-word drills will primarily improve sight-word recognition rather than reading comprehension. Topic: Competency 008

Which of the following students is most likely to benefit from an assistive technology device like a paper-based computer pen?

A. Visual learner with physical disabilities B. Auditory learner with social skills deficits C. Nonverbal learner with behavioral challenges D. Tactile learner with memory challenges (D) The correct response is (D). A paper-based computer pen allows students to take notes and record the teacher's words at the same time. This is a great tool for tactile learners who learn through fine motor activities like writing but who also have memory issues. When the student uses the pen to review the notes, he or she can hear the recording of the classroom discussion simultaneously. (A) is not correct because a visual learner with physical disabilities may not be able to hold the pen and is better served with a multimedia presentation that features visuals. (B) is not correct because an auditory learner with social-skills deficits is likely to remember discussion and lectures but needs a structured approach for interacting with peers. (C) is incorrect because a nonverbal learner with behavioral challenges will not be served by the paper-based computer pen. Generally, nonverbal learners need more hands-on manipulatives to demonstrate understanding since they cannot communicate understanding through speech.

A second-grade inclusion teacher arranges her student desks into small groups of four. She places her teacher desk on the right side of the class and an oval table on the left side. Around the oval table, she places four student chairs. She has a teacher chair on one side of the table. On the wall, she places a poster with the following words: Presenter, Researcher, Note Taker, and Discussion Leader. Given this classroom arrangement, which of the following types of instruction would you expect to see if you visited?

A. Whole-group instruction and leveled groups B. Cooperative learning and targeted small-group instruction C. Independent study and project-based learning D. Flexible grouping and sustained silent reading (B) The correct response is (B). Given the grouping of the desks and the poster specifying individual roles within each group, this classroom is organized for cooperative learning. Furthermore, the oval table on the side with four student chairs and one teacher chair indicates small-group targeted instruction. (A) is not the best answer because there is no indication of all desks facing one direction, and the roles on the poster do not mesh with leveled groups. (C) is not the best answer because independent study and project-based learning would have more individual spaces, like study carrels and work spaces with supplies for projects. (D) is not the best answer because, while there is a sense with the table to the side that the teacher may do some flexible grouping, there is no evidence of an abundance of books and plans for sustained silent reading.

A special education teacher is administering a fluency assessment to a sixth grader. The teacher notes that the student reads 110 words per minute, which is below grade-level expectations. The area of fluency that the teacher is assessing is

A. accuracy.B. expression.C. syntax.D. rate. (D) The correct response is (D). Rate is the speed at which a student reads, measured in words per minute. Typically, a sixth-grade student should be reading 120-150 words per minute. (A) and (B) are incorrect because they are different aspects of fluency, related to the number of mistakes a student makes while reading and the tone and emphasis the student uses. (C) is incorrect because syntax refers to the complexity level of the sentences a student speaks or writes.

The most commonly diagnosed behavioral disorder in children is

A. autism. B. ADHD. C. schizophrenia. D. depression. (B) The correct response is (B), as roughly 6 percent to 9 percent of children are diagnosed with ADHD. (A) is incorrect because a lower rate of diagnosis is found for autism. (C) and (D) are incorrect because schizophrenia and depression are not classified as behavioral disorders.

Research on effective teaching indicates that students who struggle with math respond positively to mathematics instruction that focuses on

A. improving basic math processing skills. B. problem-solving. C. conceptual understanding. D. all of the above. (D) The correct response is (D). Evidence-based studies in mathematics instructional strategies have shown positive results from a group of methods that focus on improving math processing skills, problem solving, and conceptual understanding. The key to solid math instruction is to use all of these strategies, not limit instruction to any one area. (A), (B), and (C) are all incorrect because they feature only one area of focus. The correct answer is all three of them in a balanced approach.

In order to build students' fluency, an appropriate prereading strategy would be to

A. have students stop at the end of each page and summarize what happened. B. have students turn to a partner and talk about what they liked and didn't like in the story. C. have students scan the text and then predict what the text will be about. D. have the students look at the title and then name similar titles from other books. (C) The correct response is (C). The act of scanning or skimming a text allows a reader to get a sense of the topic and words before actually reading. Students use this knowledge to make their predictions and build interest in the text. This prereading strategy builds fluency by helping students feel more prepared for the words they encounter. (A) is incorrect because it describes a strategy that is used during reading, and it actually works against fluency by requiring the student to pause or stop. (B) is incorrect because it is an after-reading strategy. (D) is incorrect because it activates prior knowledge but does not expose students to the text itself. It is the opportunity to skim the words in the text that builds confidence and fluency.

A fourth-grade student who recently immigrated to the United States with her family never looks her teacher in the eye. The teacher, after several months of documenting this behavior, refers the student for evaluation by the district's autism team. An important factor for the campus special education review team to consider is that

A. lack of eye contact is a sign of an autism spectrum disorder. B. autism crosses cultural and linguistic borders. C. it is overwhelming to be an immigrant and that such behaviors are to be expected. D. cultural norms and expectations impact student behavior and teacher interpretation. (D) The correct response is (D). In some cultures, it is disrespectful to look an authority figure in the eye. This may well be a factor in the scenario described. It is critical that special education teachers analyze how culturally and/or linguistically diverse backgrounds can impact behavior. (A) is incorrect because, while lack of eye contact can be a sign of an ASD, it can also be a sign of other causes, such as cultural expectations or depression. (B) is incorrect because, while autism does cross all cultural and linguistic borders, that is not the issue that led to this particular referral. (C) is incorrect because the issue here is whether the student has displayed enough potential signs of ASD to warrant a special education referral for testing.

A special education teacher working with a student with multiple disabilities moves the student to be part of the class's circle time as part of the student's IEP. The teacher can ensure her own safety and the safety of the student during the lift and transfer by

A. lifting with her legs, not her back. B. bending at the waist and moving the student quickly. C. practicing with each student in her class. D. leaving the student in his or her wheelchair, even though he or she is apart from the other students sitting on the floor for circle time. (A) The correct response is (A). Applying knowledge of appropriate body mechanics to ensure safety in transfer, lifting, positioning, and seating is part of Competency 004. Special education teachers receive training in the proper way to lift students. To prevent injury, it is essential to bend and lift with the legs while maintaining a straight back. (B) is not correct because bending at the waist and lifting with the back can cause serious injury. (C) is not correct because there is no reason to lift a student unless instructed to do so by the student's IEP or because the situation requires it. (D) is incorrect because the student's IEP indicates the need to include the student fully in circle time, and the IEP must be followed by law.

According to the Council for Exceptional Children's code of ethics, special educators and related professionals are bound to

A. maintaining challenging expectations for individuals with exceptionalities. B. using evidence, instructional data, research, and professional knowledge to inform practice. C. advocating for professional conditions and resources that will improve learning outcomes of individuals with exceptionalities. D. all of the above. (D) The correct response is (D). Maintaining challenging expectations for individuals with exceptionalities; using evidence, instructional data, research, and professional knowledge to inform practice; and advocating for professional conditions and resources that will improve learning outcomes of individuals with exceptionalities are all part of CEC's code of ethics for special education professionals. (A), (B), and (C) are incorrect because, although they are all within the code of ethics, none of these answers are correct as single answers.

A five-year-old child and his parents have just moved into the school district's boundaries. The parents bring the child to the school to register for kindergarten and explain that he is not yet speaking. The school psychologist immediately begins the paperwork to refer the child for special education evaluation. In selecting assessments for this child to complete, the psychologist and test administrators must keep in mind that

A. only paper-and-pencil tests are accepted in determining eligibility. B. tests must be selected and administered so that the child's inability to speak does not interfere with the assessment of the child's aptitude or achievement level, if at all possible. C. there must be an interpreter present during testing who can communicate with the child using sign language. D. the child is only five and will likely learn to speak eventually. (B) The correct response is (B). When administering diagnostic assessments for special education eligibility, diagnosticians and school psychologists must seek out assessments that ensure a student's disability or first language do not interfere with the assessment of the student's aptitude or achievement level where possible. For a child who does not speak, this means that the diagnostic tests must give the child an opportunity to demonstrate aptitude without the necessity of a verbal response. (A) is incorrect because a variety of tests are accepted in determining eligibility, including checklists and observational assessments. (C) is incorrect because the child is nonverbal, not deaf or hard of hearing. Unless the child and his family indicate that sign language is the child's primary means of communication, an interpreter is not necessary. (D) is incorrect because a child who is not speaking at age 5 is demonstrating severe developmental delays. Special education testing is definitely warranted.

The primary advantage of Mr. Aquino's summarization framework is to

A. prevent students from listing all the details of a story in place of a summary. B. give students a summarization strategy they can use with all texts. C. prevent students from cheating. D. ensure that all students will come up with the same answer. (A) The correct response is (A). When students are asked to summarize, they frequently make the mistake of just retelling all the high points in the story. The primary advantage of a framework like the one Mr. Aquino uses is that it gives a firm structure for students to build a summary that focuses on the gist of the story, not the details. (B) is not correct because the "Who-Wanted-But-So" strategy only applies to fiction. (C) is not correct because the use of a summarization framework does not specifically prevent cheating. (D) is not correct because, while the frame (Who Wanted But So) is set, the way that frame is filled in is entirely up to the student.

A fifth-grade student with learning disabilities consistently procrastinates in completing his assignments and is in danger of failing two subjects due to incomplete assignments. The student's teacher has just assigned the class a research paper that will weigh heavily on the language arts grade. An appropriate behavioral intervention for this student would be to

A. provide a daily planner and map out partial completion dates as well as the final completion date. B. review the guidelines of the assignment and have the student sign his name, indicating he understands the due date and the consequences of not completing the assignment. C. have the student stay in the classroom during recess every day to write the paper. D. promise the student a candy bar for every paragraph he writes. (A) The correct response is (A). When a student's procrastination behaviors interfere with academic performance, the procrastination must be addressed in as proactive and positive a way as possible. Teaching a student how to use a planner and setting smaller deadlines leading up to the final deadline will help the student both in school and beyond. (B) is incorrect because having the student acknowledge the deadline does not actually address the cause of the procrastination or provide a path to change the behavior. (C) is not the correct answer because it is punitive and it assumes the student will miss the deadline. It does not reflect confidence in the student. (D) is incorrect because extrinsic rewards are only good in the moment. Change occurs when students respond to intrinsic rewards. In addition, candy bars are not nutritious and should not be handed out to students as "bribes" for completing work.

A first-grade teacher sets up several math stations for students to rotate through in groups of four to five. The teacher sits at a large table that serves as one station and assigns students to work in pairs or groups of three to represent and solve addition problems using plastic teddy bears. As students solve the problems, the teacher observes and immediately points out mistakes so that students can try again. The teacher praises each pair or small group upon successful completion of the problem. The instructional approach that this teacher is utilizing is

A. teacher-directed cooperative learning. B. visual discrimination. C. generic problem solving. D. verbalization. (A) The correct response is (A). Teacher-directed cooperative learning is a targeted instructional approach that involves students working in small cooperative groupings while the teacher is nearby and able to observe student strategies and provide direct and immediate feedback. (B) is incorrect because visual discrimination involves presenting students with two or more symbols or shapes and having them distinguish between them based on their visual differences. (C) is incorrect because generic problem solving involves giving students strategies like highlighting keywords to use every time they encounter certain problems. (D) is incorrect because verbalization involves talking through each step in a problem-solving process so that the student and others can hear what the student is thinking.

Two co-teachers choose to use the "One Teach, One Observe" model for a lesson on volcanoes. What would this model look like in action?

. A. One teacher would pretend to be a reporter on location at a volcanic eruption, reporting what she sees, while the other teacher presents the normal progression of a volcanic eruption. B. One teacher presents the normal progression of a volcanic eruption, using visuals and 3-D models, while the other teacher watches the students in the class and keeps notes on their levels of participation and conceptual understanding. C. One teacher presents information to the boys while the other teacher presents the same information to the girls. D. One teacher sets up a station with a model of an erupting volcano and the other teacher sets up an interactive whiteboard activity that involves labeling the parts of a volcano. Students rotate between the stations. (B) The correct response is (B). The "One Teach, One Observe" co-teaching model is ideal for gathering informal observational data on students. While one teacher presents the lesson, the other teacher circulates around the room and makes notes about students' behavior, level of participation, and level of conceptual understanding. (A) is incorrect because it describes a team teaching model. (C) is incorrect because it describes a parallel teaching model. (D) is incorrect because it describes a station-teaching model.

A container of milk is 3/8 full. What percentage of the container is full?

A. 0.5 B. 0.38 C. 0.75 D. 0.375 (D) The correct response is (D). 3/8 converts to 0.375 in decimal form. This can be found by dividing 3 by 8 on a calculator. (A), (B), and (C) are all incorrect answers. Choosing (B) reveals a conceptual misunderstanding of how to convert fractions to decimals. The numbers in the fraction do not simply transfer over to the decimal form.

Use the information below to answer questions 15 and 16.A teacher presents students with the following number pattern:1, 3, 9, 27, __Given this pattern, the next number would be which of the following?

A. 81 B. 45 C. 54 D. 1 (A) The correct response is (A). The key to this number pattern is that each number is multiplied by 3 to calculate the next number. The final given number is 27, and since 27 multiplied by 3 equals 81, (A) is the correct answer. (B) and (C) are incorrect because while they are multiples of 3, they are not the correct answer to 27 × 3. (D) is incorrect because it makes the assumption that the pattern is repeating when there is no evidence of this.

Which of the following examples would best serve as a transition for students moving from a concrete to an abstract conceptual understanding of fractions?

A. A pie cut into eight pieces B. A pie graph divided into four parts with one part shaded C. An apple cut into 12 slices D. A recipe requiring several ingredients in half- and quarter-cup or teaspoon measurements (B) The correct response is (B). The transitional stage of conceptual development involves visual or pictorial representations of objects, so a pie graph is the appropriate choice. (A) and (C) are incorrect because both the pie and the apple are concrete examples, the kind useful for initial conceptual development. (D) is incorrect because a recipe would provide a way to apply a student's understanding of fractions, but it would not be the transitional phase of concept development.

Which of the following curriculum development projects would a special education teacher be most likely to work with an SLP to develop?

A. A rubric for an oral presentation projectB. Calculator practice activitiesC. A fast-food restaurant simulation, practicing how to count moneyD. A science fair project (A) The correct response is (A). An SLP is a Speech-Language Pathologist, one of the related-services colleagues a special education teacher will have the opportunity to work with. An oral presentation project is a wonderful opportunity to develop students' expressive-language skills, and this is an area in which an SLP will have specific criteria they expect to see. Creating a rubric to assess the project will provide an opportunity to target specific expressive language skills using the SLP's criteria. (B) and (C) are incorrect because an SLP's focus is communication and language skills, not mathematics skills. (D) is incorrect because an SLP would not be focused on science content.

What should precede the development of a behavior intervention plan?

A. A tour of the student's home environment B. Functional behavioral assessment C. Review of the discipline policies of the student's teachers D. Review of the student's STAAR scores (B) The correct response is (B). Prior to writing a behavior intervention plan, the IEP team should collect data for a functional behavioral assessment in order to identify the cause of the behavior, its frequency, and situations where the student does not exhibit the behavior. Having this data helps the team establish positive supports to redirect and reduce the inappropriate behavior. (A) is incorrect because a tour of the home environment is not necessary when the BIP is addressing inappropriate behaviors at school that interfere with learning. (C) is incorrect because, while reviewing classroom discipline policies may provide useful information in terms of how the student reacts to various disciplinary approaches, it is not a complete picture of the student and the causes of the behavior. (D) is incorrect because STAAR results do not reveal the central cause of the inappropriate behavior.

A special education teacher working with students with intellectual disabilities and autism is teaching one-to-one correspondence. The most appropriate instructional tool for the teacher to use in teaching this concept would be which of the following?

A. A worksheet with symbol-supported textB. A graphic organizerC. Blocks of different colorsD. A calculator (C) The correct response is (C). Students with intellectual disabilities need concepts to be as concrete as possible. Providing manipulatives like colored blocks allows students to place each block as they identify it by number (for example, one block, two blocks). (A) is incorrect because a worksheet is not concrete enough for initial concept development. Symbol-supported text is an accepted way to help nonreaders access content, but the activity itself is still a worksheet. (B) is incorrect because a graphic organizer is a visual representation, and this is a transitional stage in concept development. Initial concept development needs to be more concrete. (D) is incorrect because a calculator is a useful tool for computation, but it is not a tool to teach the concept of the number 1 being equivalent to one object, which is the essence of one-to-one correspondence.

Which of the following transition goals meets IDEA 2004 requirements?

A. After high school, Alex will learn janitorial skills for placement in a supported employment position in the local community. B. In grade 11, Tina will pass the STAAR Alt. C. Juan will apply to community colleges and major in art. D. After high school, Mia will live at home. (A) The correct response is (A). IDEA 2004 requires that transition goals be "results-oriented" and measurable. The goal should state the intended outcome for the year after the student graduates from high school. Only (A) meets all of these criteria. (B) is incorrect because it is a goal for the student during high school. Transition goals must focus on postsecondary outcomes. (C) is incorrect because it does not state a time by which the outcome is expected. (D) is incorrect because it does not include employment and education/training. These are expected to be part of any postsecondary goal.Topic: Competency 007

Use the information below to answer questions 15 and 16.A teacher presents students with the following number pattern:1, 3, 9, 27, __What type of number pattern is this?

A. Arithmetic sequence B. Fibonacci sequence C. Harmonic sequence D. Geometric sequence (D) The correct response is (D). A geometric sequence is made by multiplying by a given value each time. This number sequence has a factor of 3. The pattern is continued by multiplying by 3 each time. (A) is incorrect because an arithmetic sequence is made by adding a given value each time. (B) is incorrect because a Fibonacci sequence is made by adding the first two numbers to get the third and then adding the second and third numbers to get the fourth, and so on. (C) is incorrect because a harmonic sequence is a sequence of numbers whose reciprocals form an arithmetic sequence.

At what point should a special education teacher in Texas advise parents of a child with intellectual disabilities to apply for state assistance programs (for example, Community Living Assistance and Support Services)?

A. At the point when special education testing begins B. When the student graduates from high school C. As soon as the student has an official diagnosis D. When parents can no longer care for their child (C) The correct response is (C). Many of the state assistance programs for individuals with disabilities have waiting lists that can last for years. However, these lists also require an official diagnosis as a qualifier. Advise parents to call the appropriate agencies and get on the waiting lists as soon as a diagnosis is assigned. (A) is incorrect because there is not yet a diagnosis when special education testing begins (unless the child has already been diagnosed by a medical professional). (B) is incorrect because some of the lists require waiting 10 years or more before an individual can apply for assistance. Waiting until a student graduates may mean no assistance until the individual is in his or her 30s. (D) is incorrect because parents must plan as far in advance as possible so that proper care can be found when the parents are no longer able to provide it.

Which of the following is the most commonly diagnosed behavioral disorder in children?

A. Autism B. Depression C. Attention deficit hyperactivity disorder D. Oppositional disorder (C) The correct response is (C). Attention deficit hyperactivity disorders are the most commonly diagnosed behavioral disorders in children, affecting 3 to 6 percent. (A), (B), and (D) are incorrect because statistically they do not approach the number of students diagnosed with ADHD.

A parent tells you her/his child has dyscalculia. In which academic area would you anticipate the student to have difficulties?

A. Reading B. Writing C. Mathematics D. Social Studies (C) The correct response is (C). "Dyscalculia" means a serious difficulty with making calculations. It is a common term for a mathematics learning disability. Therefore, (A), (B), and (D) are all incorrect.

When formulating instructional objectives for a student's Individualized Education Program (IEP), what should the Annual Review and Dismissal (ARD) committee do?

A. Base the objectives exclusively on the grade-level TEKS. B. Construct the objectives based on what the student's parents want the child to be able to do. C. Let the student choose his or her own objectives. D. Review the student's current level of performance, and write reasonable objectives that reflect the grade-level TEKS as closely as possible and promote academic growth. (D) The correct response is (D). The ARD committee is responsible for writing and agreeing on reasonable objectives that will foster student growth and development while reflecting the grade-level TEKS as closely as possible. (A) is incorrect because the purpose of an IEP is to create an educational program that will best meet the student's needs. The TEKS may need to be modified slightly to accomplish this. (B) and (C) are incorrect because a child's IEP objectives must reflect the entire committee's decisions about how to best serve the child. No one stakeholder can dictate the content of the IEP.

Use the information below to answer questions 7 and 8.In assessing a sixth-grade student's oral reading, a special education teacher uses several fictional passages leveled from 1.0 to 6.0. The teacher places each passage in front of the student, starting with the 1.0 passage, and asks the student to read it aloud. The chart below represents the results of the assessment: Reading Level Accuracy percentage 1.0 98% 2.0 96% 3.0 95% 4.0 92% 5.0 85% 6.0 70% Two days after the assessment, the special education teacher is with the student's class in the school library. She notices the student looking at an action/adventure book. It is approximately a 6.0 reading level. An appropriate reaction on the part of the teacher would be which of the following?

A. Call the student aside and recommend a similar-themed book that is written at a 3.0 reading level.B. Let the student check out the original book.C. Walk over to the student and say, "You can't read that. It is too difficult."D. Tell the student she will use the original book for the next reading lesson they work on together. (A) The correct response is (A). A book at a 6.0 reading level is at this student's frustration level based on the assessment. This means the student will likely run across a number of words he does not know how to read. This can lead to frustration and abandoning the book. Since the special education teacher is present in the library and sees the student choose a book on his frustration level, she is able to redirect him toward a book on his independent level. He will get far more enjoyment and reading self-confidence out of a book at his independent level. (B) is incorrect because it sets the student up for a frustrating and potentially self-defeating reading experience. (C) is incorrect because it promotes a negative self-image about the student's ability to read and become a better reader. (D) is incorrect because the level 6.0 book is not on the student's instructional level; it is on the student's frustration level.

A student in an ALE classroom has autism and is nonverbal. Symbol-based communication and a picture-based daily schedule are working, but progress is slow. The student has recently shown a fascination with the classroom tablet devices. What type of assistive technology might the teacher recommend the IEP committee consider for this student?

A. Communication app for the tablet device B. Talking calculator C. Handwriting guides D. Speech-recognition program for the classroom computer (A) The correct response is (A). An ever-increasing number of apps for tablet devices and smartphones are being developed. Many of these are useful in special education. For the student described in this scenario, one of the communication apps that allows teachers to build the student's visual schedule and use the voice-over capabilities of the tablet appears to be an ideal option to motivate the student and help him progress. (B) is incorrect because the student's issue is communication, not mathematics. (C) is incorrect because the student's issue is communication, not handwriting. (D) is not the correct answer because the student is nonverbal, so speech-recognition software has no use in this scenario.

What techniques may be used to conduct a functional behavioral assessment?

A. Data analysis B. Structured interviews with current teachers C. Standardized behavior assessment D. All of the above (D) The correct response is (D). When conducting a functional behavioral assessment, data analysis, indirect assessment (including structured interviews), and direct assessment (including standardized assessments) may all be used. However, they are not all required. It is up to the IEP team what techniques they will use to gather the information they need to identify the root cause of the inappropriate behavior.

Which of the following is NOT an area in which diagnostic assessments to determine eligibility for special education services might be given?

A. DevelopmentB. Social emotional behaviorC. Career placementD. Auditory processing (C) The correct response is (C). When determining eligibility for special education services, (A) development, (B) social emotional behavior, and (D) auditory processing may all be assessed. (C) career placement is the only area that would not be part of determining eligibility for special education services.

Use the information below to answer questions 73 and 74.A third-grade teacher displays the following words on a prominent bulletin board in her classroom:acute, right, obtuse, area, perimeter, symmetry, and volume.Which of the following instructional strategies would be the best use of the mathematical vocabulary words posted on the bulletin board?

A. Divide the students in groups of three and assign one word from the bulletin board to each group. Direct the groups to illustrate the meaning of their words on pieces of chart paper. Then have the class play a game of visual charades, where a group presents their illustration to the class, and the class guesses which vocabulary word is illustrated. B. Avoid pointing out the words and let them be a natural part of the classroom environment for the students to absorb. C. Have students refer to the bulletin board when completing math assignments in order to spell words correctly. D. Have students find the same words in magazines and newspapers and staple their environmental print examples to the bulletin board. (A) The correct response is (A). Using the words as catalysts for peer interaction and discussion of the math vocabulary allows students to internalize the meanings of the words both by visualizing them and by teaching them through the modified game of charades. (B) is incorrect because math vocabulary needs to be taught explicitly to help students truly understand what the words mean. (C) is incorrect because, while the words can be useful spelling references, the power in these words is in students using them and internalizing their meanings. (D) is incorrect because while this strategy helps students recognize how math is used beyond school, it is not as effective as (A) in helping students grasp the meanings of the words.

A special education teacher wants to share study strategies with students with learning disabilities and their families. He organizes an evening hands-on workshop where he presents information about different learning styles, and models study strategies that fit each of the learning styles. What is this teacher trying to accomplish?

A. Earning a Teacher of the Year nominationB. Showing the general education teachers that the special education department does more for students than the academic departmentsC. Helping parents understand where their students have weaknessesD. Teaching students and their families how to recognize their strengths and use those strengths to achieve success in the regular education environment, no matter who the teacher is (D) The correct response is (D). One of the most important things a special education teacher can do is teach students and their families strategies the student can use to succeed on his or her own. Knowing how one learns best allows a student to take control of his or her own learning. In addition, focusing on strengths is so important with students with disabilities who often find people and themselves focusing only on their weaknesses. (A) and (B) are incorrect because the teacher is not acting for himself. (C) is incorrect because the focus of the event is on learning strengths, not weaknesses.Topic: Competency 005

Which of the following is NOT a stage of the writing process?

A. EditingB. PrewritingC. Counting the wordsD. Publishing (C) The correct response is (C). Most educators think of writing as a five-step process: prewriting, drafting, revising, editing (proofreading), and publishing. Counting the words is not a part of this process, although writers are often curious to know how many words they have written. (A), (B), and (D) are all incorrect answers because they are stages of the writing process.

When communicating with parents, what is the best form of communication in today's world?

A. Emails B. Texts C. Voice mail D. Any of the above, depending on the parents' preferences (D) The correct response is (D). Mobile technologies have expanded the forms of communication teachers can use and make communication more immediate in many cases. However, the key to positive parent communication is to find out how parents prefer to communicate. Some may have busy careers and not appreciate a phone call on their cell phone during work hours. Others may not be comfortable with texting. At the beginning of the year, teachers can establish positive two-way communication channels by asking parents to indicate their preferred method of communication. (A), (B), and (C) are all incorrect because, although they are each a valid form of communication, none of them can meet every communication need throughout the school year.Topic: Competency 012

What is the ARD committee in Texas equivalent to in federal regulations regarding special education evaluation procedures?

A. FAPE committeeB. LRE committeeC. IEP committeeD. CEC committee (C) The correct response is (C). The ARD (Admission, Review, and Dismissal) committee is the Texas equivalent to the IEP committee referenced in IDEA regulations. (A) is incorrect because FAPE stands for Free and Appropriate Public Education. (B) is incorrect because LRE stands for Least Restrictive Environment. (D) is incorrect because CEC stands for Council for Exceptional Children, a nonprofit organization that advocates for the rights of students with disabilities and the educators who serve them.

What term best describes an assessment that measures group performance against an established standard at defined points along the path toward the standard?

A. FormativeB. BenchmarkC. SummativeD. Alternative (B) The correct response is (B). A benchmark assessment measures group performance against an established standard at defined points. (A) is incorrect because formative assessments are administered during instruction to guide the teacher in instructional decisions. (C) is incorrect because summative assessments are administered at the end of units to determine student progress. (D) is incorrect because alternative assessments are given when traditional multiple-choice formats cannot accurately measure a student's knowledge or skills.

Which of the following academic difficulties may characterize a language-learning disability?

A. Frequent misspelled words B. Difficulty memorizing the multiplication tables C. Frequent missing assignments D. Difficulty working with other students in groups (A) The correct response is (A). Language-learning disabilities involve difficulties or delays with written language, grammar and spelling, and handwriting. Frequent misspelled words would be indicative of a language-learning disability. (B) is incorrect because it is related to mathematics. (C) is incorrect because it is related to either behavioral or attention-deficit issues. (D) is incorrect because it is related to social skills or behavioral issues.

Use the information below to answer questions 73 and 74.A third-grade teacher displays the following words on a prominent bulletin board in her classroom:acute, right, obtuse, area, perimeter, symmetry, and volume.Given these vocabulary words, what kind of math unit is the teacher most likely preparing to present?

A. Geometry B. Probability and statistics C. Measurement D. Number sense (C) The correct response is (C). The words displayed on the bulletin board are all math vocabulary related to measurement. The teacher is preparing students for a unit on measurement. (A) is incorrect because a unit on geometry would include terms such as the names of shapes or the types of flips. (B) is not the correct answer since a unit on probability and statistics would include terms such as "mean," "median," and "mode." (D) is incorrect because a unit on number sense would include basic math terms like counting, numbers, and number words.

A special education teacher has two paraprofessionals assigned to her class. Which of the following would be the best use of these assistants' time?

A. Have the paraprofessionals make copies and laminate materials most of the dayB. Ask the paraprofessionals to administer medications to the studentsC. Train the paraprofessionals to help facilitate the instructional programs the teacher is using so that instruction can be divided into individual and small group timeD. Have the paraprofessionals monitor the students while the teacher does required paperwork (C) The correct response is (C). Special education classrooms are meant to support individualized and targeted instruction. Paraprofessionals can be excellent instructional resources when trained in the programs students are using. (A) is incorrect because, while paraprofessionals can assist with classroom organization and preparation, their primary use should involve student instruction. (B) is incorrect because no one at a school may administer medications other than the school nurse and the school nurse's staff. (D) is incorrect because it is critical that teachers and paraprofessionals maximize instructional time by engaging with the students and working on IEP goals whenever possible. Generally, paperwork should be left for conference periods and before and after school.

Use the information below to answer questions 54 and 55. Mrs. Cullen is a sixth-grade reading teacher. She has both regular and special education students in her classroom. At the beginning of the year, she administers a variety of informal assessments and observes students' strengths and weaknesses. She then sets up stations around the classroom where students can work independently or in pairs on reading and writing tasks. Each day, for 15 minutes, Mrs. Cullen teaches a whole-group mini-lesson related to the TEKS. Then students are directed to stations. During station time, Mrs. Cullen conducts small-group targeted lessons on various reading skills. While the groups never have more than five students, the students called up each day change depending on the skill being taught. What instructional technique is Mrs. Cullen using?

A. Homogenous groupingB. Whole-group instructionC. Self-selected readingD. Flexible grouping (D) The correct response is (D). Mrs. Cullen is using flexible grouping to ensure that students receive targeted instruction in the skill areas they most need to improve. (A) is incorrect because homogenous grouping involves placing students in groups by ability and not changing the groups once established. (B) is incorrect because Mrs. Cullen has structured her classroom so that students receive whole-group, small-group, paired, and independent instruction. (C) is incorrect because Mrs. Cullen chooses the members of the small group based on data gathered at the beginning of the year.

Katie, a fourth-grade student with ADHD, is often off-task. Her regular education teacher places her at a table close to the front of the room with tablemates who are very focused on their work. What is Katie's teacher doing?

A. Ignoring Katie's behavior B. Implementing a simple accommodation to improve Katie's performance C. Encouraging Katie's classmates to be watchdogs D. Modifying Katie's assignments (B) The correct response is (B). Moving Katie closer to the front of the room and choosing tablemates who will not further distract her is a simple accommodation, which may help Katie to concentrate better on schoolwork. If this does not work, more complex accommodations would be developed. (A) is incorrect because ignoring Katie's behavior will only lead to poor grades due to incomplete assignments. (C) is incorrect because a teacher should never use students as part of their discipline plan. (D) is incorrect because Katie is capable of completing the assignments, but she becomes distracted easily because of her documented disability.

During the first two weeks of school, a first-grade teacher observes his/her students completing various assignments and takes note of their strengths and weaknesses. What kind of assessment is the teacher giving?

A. Informal B. Norm-referenced C. Summative D. Formal (A) The correct response is (A). Observations are considered informal assessments. (B) is incorrect since observations are not standardized assessments, but rather professional judgments. (C) is incorrect since the observations were conducted at the beginning of school, not at the end of a unit. (D) is clearly incorrect since observations are not considered formal.

For special education students with intensive disabilities, school districts must offer public education until age 23. Which of the following curriculum choices would be the most appropriate for students after high school to age 23?

A. Intensive reading interventionB. On-the-job coaching and independent living skillsC. Academic courses along with two electivesD. Social skills training (B) The correct response is (B). The ultimate goal of a post-high school program should be to prepare students for success in their adult life in the community. For this reason, on-the-job coaching and independent living skills should be a curriculum priority. (A) is incorrect because an intensive reading intervention takes a great deal of instructional time. At this point in school, students may still learn to read, but the focus should be on vocational and life skills. (C) is incorrect because once students have completed the typical four years of high school, they are ready for a more specialized program than the typical high school schedule. Again, the focus needs to be on vocational and life skills. (D) is incorrect because it is not comprehensive enough. Social skills are indeed important aspects of preparing students for independent living, but they are only one aspect. They should be a part of a larger vocational and independent living skills curriculum.

Which of the following behaviors would be a potential warning sign for an epileptic seizure?

A. Irritability B. Extreme hunger C. Nervous energy D. Unexplained confusion, sleepiness, or weakness (D) The correct response is (D). Epileptic seizures have a number of potential warning signs. Unexplained confusion, sleepiness, and/or weakness are among those warning signs as are strange tastes or feelings, eyes rolling up, and drooling. (A), (B), and (C) are all incorrect as they are not among the potential warning signs of an epileptic seizure. (A) irritability can be a warning sign for insulin shock for students with diabetes.

A veteran teacher advises a new colleague in the special education department to identify some strengths she sees in each of her students and to call parents in the first few weeks of school to compliment their child on these strengths. What is the primary advantage of this approach to parent-teacher communication?

A. It allows the new teacher to ignore all the problems she is going to have with students.B. It builds respect with parents who see that the teacher cares about their child and are often pleasantly surprised to be getting a "good" call.C. It can be documented on the first evaluation the new teacher receives.D. It ensures that the new teacher will believe in herself enough to make it through the school year. (B) The correct response is (B). Positive phone calls are an excellent way to build respectful relationships with parents at the beginning of the school year. For some parents, it may be the first positive phone call they have ever received from school. By showing parents that a teacher cares about and appreciates students from the beginning, it becomes easier to problem-solve together if negative behaviors emerge later in the year. (A), (C), and (D) are all incorrect as they focus on the teacher only, not on the parent-teacher relationship.

Which of the following statements is true of autism?

A. It is more common among boys than girls. B. It is more common among children born from teenage parents. C. It is more common among girls than boys. D. It is curable. (A) The correct response is (A). Autism spectrum disorders disproportionately affect boys. They are five times more common among boys (1 in 54) than among girls (1 in 252), according to the Centers for Disease Control and Prevention. Therefore, (C) is incorrect. (B) is incorrect because ASD is more common among children of older parents. (D) is incorrect because there is no known cure for autism, although early intervention and therapies can help a child overcome many of his/her communication barriers.

The parents of Jake, a fourth grader, are notified that an ARD meeting will take place the following day. At the meeting, the ARD committee presents the results of evaluations and other information suggesting that Jake needs special education services for a learning disability. What has the ARD committee done wrong?

A. Jake's parents should have been notified of the meeting at least five days in advance. B. The ARD committee should not have shared evaluation results or other confidential information .C. The idea that Jake has a learning disability should not have been broached by the ARD committee. D. Jake's parents should have convened the meeting. (A) The correct answer is (A). (B) is incorrect because an ARD committee is authorized to share confidential information about a minor child with parents. Likewise, (C) is incorrect because the ARD committee is authorized to suggest that Jake has a learning disability. (D) is incorrect because the ARD committee is authorized to convene the meeting.Topic: Competency 012

Which of the following student activities is an example of structural analysis?

A. Matching words to pictures B. Reading a 100-word passage in less than a minute C. Identifying prefixes and suffixes in a list of words D. Using a graphic organizer for prewriting (C) The correct response is (C). Structural analysis involves seeing parts of multisyllabic words in order to decode them. Prefixes and suffixes are word parts that carry a predictable meaning. Being able to see them in long words also means being able to guess at the meaning of those words. (A) is incorrect because matching words to pictures is a comprehension activity. (B) is incorrect because reading a 100-word passage in less than a minute is a fluency activity. (D) is incorrect because using a graphic organizer in writing has nothing to do with breaking a multisyllabic word into its parts.

A third-grade student is tested for learning disabilities and is found to have memory deficits. Based on this diagnosis, in which area of mathematics is this student likely to have difficulty?

A. Math facts B. Geometry C. Graphs and charts D. Fractions, decimals, and percentages (A) The correct response is (A). Memory deficits often result in math fact memory and retrieval issues. This makes completing math fact drills arduous for students who find themselves frustrated trying to retrieve a math fact they had memorized a few days earlier. (B) is incorrect because geometry issues tend to be a result of visual-spatial deficits. (C) is incorrect because issues with graphs and charts can be due to visual-spatial deficits or reading deficits. (D) is incorrect because issues with fractions, decimals, and percentages can occur for a variety of reasons. The key is identifying at what point in the process of converting fractions to decimals or decimals to percentages the student struggles.

Which of the following situations would be a good reason to provide community-based instruction?

A. Ninth-grade students do not see how algebra relates to the real worldB. Students in a high school ALE class have been practicing how to order food off a menu at a restaurantC. Students in a ninth-grade English class all passed the STAARD. Parents want to know why the district chose a particular reading program (B) The correct response is (B). Community-based instruction is frequently used in self-contained classrooms as part of preparing students to transition into the community. It makes sense to have an Alternative Learning Environment class (that has been practicing ordering food off a menu) actually go to a restaurant and order off a menu. This allows teachers to assess generalization of skills. (A) is incorrect because a teacher can provide numerous examples of how algebra relates to the real world without leaving the campus. (C) is incorrect because community-based instruction is not a reward, per se. It is a planned part of instruction for students with significant disabilities. (D) is incorrect because community-based instruction has nothing to do with curriculum selection.

A sixth-grade teacher hands out copies of a blank Venn diagram for students to use during prewriting. What form of writing are the students most likely going to be assigned?

A. Persuasive B. How-to C. Compare/contrast D. Narrative (C) The correct response is (C). A Venn diagram is composed of two circles that intersect. It is perfect for compare/contrast writing as the comparisons can be written inside the circle while the contrasts can be written outside the circle. (A), (B), and (D) are not correct as they represent forms of writing better suited to other graphic organizers.Topic: Competency 008

During a reading lesson, a first-grade teacher writes the word "boy" on the board. She points to the letter "b" and asks students to make the sound of the letter. Which of the following concepts is the teacher addressing?

A. Phonemic awarenessB. PragmaticsC. FluencyD. Letter-Sound correspondence (D) The correct response is (D). The understanding that a written symbol like the letter "b" makes a unique sound /b/ is the essence of letter-sound correspondence. (A) is incorrect because phonemic awareness is the ability to hear and recognize phonemes. It is a precursor to phonics and letter-sound correspondence. (B) is incorrect because "pragmatics" is a term for social conversation skills and is an area that speech-language pathologists address. (C) is incorrect because fluency is the ability to read with appropriate speed and good expression.

The law that mandates that states provide a free appropriate public education to all children with disabilities or risk educational funding cuts or withholdings is

A. Public Law 89-10.B. Public Law 93-112.C. Public Law 94-142.D. Public Law 94-479. (C) The correct response is (C). Public Law 94-142, better known as IDEA, mandates a free appropriate public education for all children with disabilities. States that do not adhere to these mandates risk educational funding cuts or withholdings. (A) is incorrect because Public Law 89-10 is the Elementary and Secondary Education Act. (B) is incorrect because Public Law 93-112 is the Rehabilitation Act, and its Section 504 is an unfunded mandate. (D) is incorrect because Public Law 94-479 gave George Washington the title of General of the United States armies posthumously.

What details must a written notice of an ARD committee meeting include?

A. Purpose, time, and place of the meetingB. Three choices for the date and time of the meetingC. Evaluation resultsD. The proposed IEP (A) The correct response is (A). A written notice for an ARD (Admission, Review, and Dismissal) committee meeting must include the purpose, time, and place of the meeting. (B) is incorrect because the written notice can specify one time and place. It does not have to offer choices. If parents cannot make that time, they may request that the ARD be rescheduled. (C) is incorrect because evaluation results are shared at the committee meeting, not included in the written notice. (D) is incorrect because the Individual Education Program is written during the ARD committee meeting and agreed upon by all members present.

Use the information below to answer questions 7 and 8.In assessing a sixth-grade student's oral reading, a special education teacher uses several fictional passages leveled from 1.0 to 6.0. The teacher places each passage in front of the student, starting with the 1.0 passage, and asks the student to read it aloud. The chart below represents the results of the assessment: Reading Level Accuracy percentage 1.0 98% 2.0 96% 3.0 95% 4.0 92% 5.0 85% 6.0 70% Based on the results, what level text would be the most appropriate to use for guided reading instruction?

A. Reading Level 2.0 B. Reading Level 4.0 C. Reading Level 5.0 D. Reading Level 6.0 (B) The correct response is (B). A student's instructional reading level is considered to be at 90 percent accuracy or one mistake out of every 10 words; 92 percent accuracy is the closest number to 90 percent and falls into the instructional to independent reading range. (A) is incorrect because 96 percent accuracy would mean that reading level 2.0 is an independent reading level. Any percentage 95 percent or higher is considered to be in a student's independent reading range. (C) and (D) are incorrect because reading levels 5.0 and 6.0 are both below 90 percent accuracy and are therefore considered to be at the student's frustration level.Topic: Competency 008

Researchers look at the strategies successful readers use to help them identify reading strategies to teach explicitly to struggling readers. Which of the following strategies is an example of a strategy most successful readers use during reading, often without consciously thinking about it?

A. Reading just to get to the end of the assignment B. Skipping over words they don't recognize C. Reading fiction and nonfiction the same way D. Pausing every few paragraphs and mentally summarizing what just happened in the text (D) The correct response is (D). Successful readers frequently check for understanding as they read a text. Often, they don't even realize they are doing so because it is such a quick mental review of what is happening and an evaluation of whether it makes sense in the context of the passage. (A) is incorrect because reading just to get to the end of the assignment is something struggling readers do. Successful readers read with a purpose. (B) is incorrect because skipping over unrecognized words is something struggling readers do. Successful readers pause and use context clues to determine the meaning of unknown words. (C) is incorrect because reading fiction and nonfiction the same way is something struggling readers do. Successful readers identify the type of text they are about to read and apply appropriate strategies.

Which reauthorization of IDEA expanded the definition of disabilities to include children with developmental delays between the ages of three and nine?

A. Reauthorization of 1990 B. Reauthorization of 1997 C. Reauthorization of 2004 D. Reauthorization of 2009 (B) The correct response is (B). The 1997 Reauthorization of IDEA expanded the definition of disabilities to include children with developmental delays between the ages of three and nine. (A) is incorrect because the 1990 reauthorization is known for the renaming of Public Law 94-142 to IDEA. (C) is incorrect because the 2004 reauthorization is known for its changes to transition and disciplinary requirements. (D) is incorrect because there was no reauthorization in 2009; instead, there was inclusion of special education in the distribution of ARRA funds (better known as stimulus funds). Topic: Competency 010

Which reauthorization of IDEA implemented a timeframe of not more than 60 days to conduct initial evaluations for special education eligibility?

A. Reauthorization of 1990B. Reauthorization of 1997C. Reauthorization of 2004D. Reauthorization of 2009 (C) The correct response is (C). The IDEA Reauthorization of 2004 includes a timeline of 60 days to conduct initial evaluations for special education eligibility. (A) and (B) are incorrect because these reauthorizations did not implement this timeline requirement. (D) is incorrect because there was no reauthorization in 2009.

Ms. Silva, a co-teacher in a second-grade classroom, works with a group of three students. She reads a sentence out loud and then has the students repeat the sentence. She instructs them to "mimic" her as closely as possible because she wants them to match her speed and expression. What modeling method is Ms. Silva using?

A. Recorded booksB. Neurological impressC. Choral readingD. Echo reading (D) The correct response is (D). Echo reading is a modeling method in which a fluent reader takes the lead and reads a sentence or line from a poem and then a developing reader repeats the sentence or line, using the same expression and speed if possible. Like all modeling methods, it provides a level of security for a struggling reader because the fluent reader pronounces all the words before the struggling student has to do so. (A) is incorrect because recorded books are digital recordings of someone reading a book so that students can listen as they follow along. (B) is incorrect because neurological impress is an intervention-modeling method where the teacher whispers the words in the student's ear simultaneously with the student's oral reading. (C) is incorrect because choral reading is when three or more students read text together out loud. It does not include modeling before the student reads, but it does provide peer support.

As part of reading instruction, a special education teacher in a self-contained classroom has a student say a word, trace a word, identify a word in text, and complete a worksheet by matching the word to a picture. Which research-based instructional strategy is being demonstrated in this teacher's classroom?

A. Repetition and review B. Computer-based instruction C. Positive reinforcement D. Visual teaching (A) The correct response is (A). By having the student practice the word repeatedly but through different activities, the teacher makes the most of the power of repetition for students with intellectual disabilities. These students often experience retention difficulties and need strategies that provide multiple opportunities to practice without allowing the student to become bored. (B) is incorrect because the students do not use a computer in this instructional scenario. (C) is incorrect because there is no mention of the teacher continuously praising the students as they complete the tasks successfully. (D) is incorrect because the activities represent multiple modalities, not just visual.

Ms. Tejeda, a special education teacher, is assigned to co-teach several classes with an eighth-grade algebra teacher. Which of the following interactions would be the most effective way for Ms. Tejeda to ensure successful collaboration?

A. Schedule a time before or after school twice a week to plan lessons together.B. Invite the algebra teacher out for appetizers and drinks to "get to know one another."C. Gather all the special education resources she uses most and drop them off on the algebra teacher's desk.D. Offer to modify all of the algebra teacher's tests for the students with special needs. (A) The correct response is (A). When working with a co-teacher, mutual lesson planning is essential to a successful collaboration. Scheduling a time to plan together twice a week and consistently being on time to these planning meetings will ensure that both teachers feel they are being respected and allowed to contribute ideas. (B) is incorrect because it implies that the co-teaching relationship is a casual friendship rather than a collegial collaboration. (C) is incorrect because it places the regular classroom teacher in the position of needing to learn "special education" techniques and does not honor the knowledge the classroom teacher brings to the collaboration. (D) is incorrect because offering to modify assessments without first co-planning and agreeing upon key points in the content gives the impression that the co-teacher's role is just to make everything easier for the students with IEPs regardless of the algebra teacher's goals for instruction.

Use the information below to answer questions 28 and 29.An elementary ALE teacher uses puff paint and index cards to create flash cards with raised words. The words on the cards are: and, is, what, I, am, to, have, see, the, you. The teacher gives each student several minutes to trace the raised words with their fingers. If the student is verbal, the teacher has the student say the word while looking at it and tracing it. If the student is nonverbal, the teacher says the word while the student looks at the word and traces it. What kind of words has the teacher selected for this activity?

A. Second-grade words B. Sight words C. Decodable words D. Academic vocabulary (B) The correct response is (B). All of the words listed are sight words. In fact, all 10 are from the Dolch sight-word list. (A) is not correct because the words are prekindergarten and kindergarten words, according to the commonly referenced Dolch grade-level sight-word lists. (C) is not correct because sight words are often not easily decodable as many are exceptions to standard phonetic rules. (D) is not correct because academic vocabulary is the language of a content area, such as "geometry" in mathematics or "summary" in reading.

The first piece of federal legislation to address the civil rights of all people with disabilities was

A. Section 504 of the Rehabilitation Act.B. the Individuals with Disabilities Education Act.C. the Elementary and Secondary Education Act.D. the No Child Left Behind Act. (A) The correct response is (A). Section 504 of the Rehabilitation Act, passed in 1973, was the first piece of federal legislation to address the civil rights of all people with disabilities. Previous federal legislation existed for people with learning disabilities, but not for students with all disabilities. (B) is incorrect because IDEA was passed two years after Section 504. (C) is incorrect because the Elementary and Secondary Education Act did not originally address the civil rights of students with disabilities. (D) is incorrect because No Child Left Behind was the 2001 reauthorization of ESEA.Topic: Competency 010

Use the information below to answer questions 54 and 55. Mrs. Cullen is a sixth-grade reading teacher. She has both regular and special education students in her classroom. At the beginning of the year, she administers a variety of informal assessments and observes students' strengths and weaknesses. She then sets up stations around the classroom where students can work independently or in pairs on reading and writing tasks. Each day, for 15 minutes, Mrs. Cullen teaches a whole-group mini-lesson related to the TEKS. Then students are directed to stations. During station time, Mrs. Cullen conducts small-group targeted lessons on various reading skills. While the groups never have more than five students, the students called up each day change depending on the skill being taught. Based on the description of Mrs. Cullen's beginning-of-the-year procedures, which of the following statements most likely describes her philosophy?

A. Special education students need to be taught differently.B. Teaching to the state standardized test ensures all students will pass.C. Data should drive instruction.D. There is only one way to teach reading. (C) The correct response is (C). Mrs. Cullen's decision to administer a variety of informal assessments at the beginning of the year shows that she values data when planning instruction. (A) is incorrect because Mrs. Cullen has set up a classroom where all students can thrive regardless of disability. (B) is incorrect because Mrs. Cullen does not appear to be focusing on test preparation. Instead, she is targeting students' skill gaps as she works with small groups. (D) is incorrect because Mrs. Cullen has set up a classroom that allows for multiple reading opportunities and represents varied ways to practice reading skills.

Which of the following is NOT a strategy that will help a teacher conduct productive and positive parent-teacher meetings?

A. State at the beginning of the meeting that the teacher has only 10 minutes and will have to end the meeting as soon as those 10 minutes are up. B. Develop a clear purpose for the meeting, state the purpose at the beginning of the meeting, and summarize how this purpose was accomplished at the end of the meeting .C. Start the meeting by noting some of the positive aspects of the child's performance and behavior. D. Schedule the meeting time around the parents' schedule needs. (A) The correct response is (A). Limiting a parent meeting to 10 minutes and making it clear that the meeting is not the teacher's top priority is a sure way to put parents in a defensive position. Such a short time span makes it difficult to have time for parent questions or for teacher-parent brainstorming and problem solving. When scheduling parent meetings, scheduling a minimum of 20 minutes and being available in case the meeting runs longer will go a long way toward building positive interactions. (B), (C), and (D) are all strategies to promote positive parent-teacher meetings and are, therefore, incorrect.

Which of the following is NOT a strategy that will help a teacher conduct productive and positive parent-teacher meetings?

A. State at the beginning of the meeting that the teacher has only 10 minutes and will have to end the meeting as soon as those 10 minutes are up.B. Develop a clear purpose for the meeting, state the purpose at the beginning of the meeting, and summarize how this purpose was accomplished at the end of the meeting.C. Start the meeting by noting some of the positive aspects of the child's performance and behavior.D. Schedule the meeting time around the parents' schedule needs. (A) The correct response is (A). Limiting a parent meeting to 10 minutes and making it clear that the meeting is not the teacher's top priority is a sure way to put parents in a defensive position. Such a short time span makes it difficult to have time for parent questions or for teacher-parent brainstorming and problem solving. When scheduling parent meetings, scheduling a minimum of 20 minutes and being available in case the meeting runs longer will go a long way toward building positive interactions. (B), (C), and (D) are all strategies to promote positive parent-teacher meetings and are, therefore, incorrect.

A middle-school teacher believes in incorporating movement to help maintain student focus and engagement. Which of the following classroom situations would be an example of that philosophy?

A. Students are assigned seats and expected to stay in their seats unless they raise a hand asking for permission to sharpen a pencil or go to the restroom. B. Each student is given an index card and asked to write a question he or she has about the lesson the teacher just presented. The teacher then has the students stand and find partners who are wearing the same color. Once all the students are standing next to their partners, the teacher asks them to exchange cards and answer each other's questions. C. The teacher gives any student who answers a question correctly a high-five. D. Students are asked to draw what they see in their heads as a teacher reads an article aloud to them. (B) The correct response is (B). The teacher's activity in (B) provides for purposeful movement, meeting the needs of kinesthetic learners while also focusing students on the content just taught. (A) is incorrect because students are not encouraged to move at all in this scenario. (C) is incorrect because, while it features positive reinforcement, the movement is all on the teacher's, not on the student's, part. (D) is incorrect because it describes the strategy of visualization, not movement.

For which students was the STAAR Alternate designed?

A. Students with learning disabilities B. Students in alternative schools C. Students with ADHD D. Students with significant cognitive disabilities (D) The correct response is (D). The STAAR Alternate was designed for approximately 1 percent of Texas schoolchildren, specifically those students with such significant cognitive disabilities that are not able to be tested fairly or accurately using a traditional multiple-choice format. Therefore, (A), (B), and (C) are all incorrect.

An initial ARD is held for a student evaluated for special education services. The student's parents gave permission for the evaluation, but during the ARD committee meeting, they talk about their concerns regarding special education labels and ultimately refuse to sign their agreement to the special education placement. The classroom teacher feels strongly that the child needs special education services and requests to have a special education co-teacher work with the student one-on-one "off-the-record." What is the appropriate response to this suggestion?

A. The classroom teacher should be fired for violating the Texas Code of Ethics for Educators.B. Special education laws require parental consent. The student must be served in the regular classroom for now.C. The classroom teacher should call the parents and let them know her plans.D. The school district will file an appeal to an impartial hearing officer. In the meantime, the child will receive special education services per the teacher's request. (B) The correct response is (B). Parental consent must be obtained before a student with a disability can be placed in a school's special education program. If parental consent cannot be obtained, a school district may appeal to an impartial hearing officer. A classroom teacher cannot make a decision to ignore this requirement. In the scenario above, the student must be served in the regular classroom for now. (A) is incorrect because the classroom teacher has not technically violated the Texas Code of Ethics by making her suggestion. As long as she does not actually carry through with her idea, firing the teacher is not the answer. However, educating her on special education law is critical. (C) is incorrect because the parents have already refused to give consent. Further discussion on this matter must take place in the context of the ARD committee or the campus administration. (D) is incorrect because when a school district files an appeal to an impartial hearing officer, IDEA provisions state clearly that the student must stay in place in the current placement until the hearing process is concluded.

To fulfill the IDEA requirement of least restrictive environment (LRE), which of the following areas must be documented in a special education student's IEP?

A. The name of the student's special education teacherB. The list of medications the student is takingC. The extent to which the student will not participate with nondisabled students in regular classes or nonacademic activitiesD. The parents' description of the student's home environment (C) The correct response is (C). The extent to which a student will not participate with nondisabled students in regular classes or nonacademic activities must be clearly documented in the student's IEP. This documentation is necessary to ensure that the ARD committee has addressed opportunities for inclusion and has documented decisions to limit inclusion for educationally sound reasons. This documentation is all part of fulfilling the least restrictive environment requirement of IDEA. (A), (B), and (D) are all incorrect as none of those are required for documentation or related to least restrictive environment.

A second grader with autism who is nonverbal gets upset in the cafeteria at lunch and lies down on the ground and begins screaming. This goes on for several minutes, disrupting lunch. Which of the following is NOT a potential cause of this behavior?

A. The noise and confusion in the cafeteria became overwhelming .B. The child's scheduled lunchtime is normally an hour later and today is a change of routine. C. The second grader wants everyone to look at him. D. The child's favorite type of chip was not in his lunch bag today. (C) The correct response is (C). Individuals with autism lack "theory of mind," the ability to recognize that other people have different perspectives. They do not have tantrums because they want to be the center of attention or want to manipulate others. Instead, they act out in frustration because they cannot communicate appropriately how they are feeling, and their frustration builds to the breaking point. (A) is a potential cause of the behavior because environmental noise can overwhelm a child with autism who lacks the ability to filter it out and turn it into background noise. (B) is a potential cause of the behavior because children with autism are extremely routine-oriented. Disruptions in the routine can seem "wrong" to the extent that the child acts out in frustration. (D) is a potential cause of behavior for similar reasons. Children with autism often develop limited eating patterns where they only eat certain foods. Changes in these foods can cause mounting frustration, which can then lead to a meltdown.Topic: Competency 006

Why were the Rehabilitation Act of 1973 and Public Law 94-142, passed by Congress in 1975, so important for students with disabilities in the United States?

A. These two laws showed the rest of the world that the United States was committed to special education. B. Prior to these two pieces of federal legislation, individual states were not offering public education to the majority of children with disabilities. C. These two laws created a designated day to honor individuals with disabilities. D. These two laws made provisions for the federal government to fund over half of the special education expenses incurred by school districts. (B) The correct response is (B). Prior to the Rehabilitation Act of 1973 and Public Law 94-142 (later named the Individuals with Disabilities Education Act), the number of students with disabilities receiving free public education was quite low across the United States. These two pieces of federal legislation opened educational opportunities for all students with disabilities by requiring states to offer a free and appropriate public education to all students. (A) is incorrect because the purpose of the federal laws was to change special education practices within the states. However, since that time these laws have indeed helped the United States emerge as one of the premier countries for special education programs. (C) is incorrect because these laws did not designate a national day to honor people with disabilities. (D) is incorrect because the Rehabilitation Act provided no funding, and Public Law 94-142 promised some federal funds for special education but has never come close to supplying 50 percent of the costs incurred by state and local education agencies.

A seventh-grade inclusion teacher uses the interactive whiteboard to model persuasive writing for her students. She writes a persuasive essay on why school should be year-round. As she writes, she comments on her word choices and talks about points to make, expanding some of them and discarding others. The instructional strategy that this teacher is using is which of the following?

A. Think-aloud B. Graphic organizer C. Language experience D. Formula writing (A) The correct response is (A). The teacher is modeling a think-aloud, a technique where the facilitator literally thinks out loud the kinds of thoughts that writers or readers think subconsciously during the process of writing or reading. Think-alouds are great strategies for helping students realize that all writers have moments of self-doubt. They are also great opportunities to reveal how writers organize thoughts in their head before writing them down. (B) is not correct because a graphic organizer is a visual planning tool. (C) is an incorrect answer because language experience is a reading method that integrates writing and reading. (D) is not correct because the teacher is not using a formula constructed to help students achieve on standardized writing tests.

Mr. Martinez, an eighth-grade social studies teacher, begins a unit on Westward Expansion in the United States by dividing students into "families" and giving each family a destination to reach in the West. Each "family" receives an index card with a list of supplies and transportation. "Family" members must talk together to figure out how they will reach their destination. What is the most likely reason Mr. Martinez begins the unit this way?

A. To build student engagement in the topicB. To prepare students for the testC. To foster healthy competitionD. To make decisions based on data (A) The correct response is (A). Mr. Martinez uses the simulation of how it felt to be a family heading west to build anticipation and engagement in the upcoming unit. (B) is incorrect because Mr. Martinez is only kicking off the unit; he has not yet covered the content that will be tested. (C) is incorrect because the activity is cooperative, not competitive. (D) is incorrect because Mr. Martinez is not using data to conduct the activity. He is allowing students to make their own decisions about how their "family" will move out West. This builds ownership.Topic: Competency 003

Which of the following choices is NOT one of the purposes of a transition assessment?

A. To identify a student's interests and vocational abilities B. To identify a student's level of self-determination skills C. To determine whether a student will pass high school D. To identify a student's strengths, abilities, and potential deficits (C) The correct response is (C). A transition assessment is built to help educators guide students to postschool jobs and activities that are likely to be most fulfilling to them. It is not used to determine whether a student graduates from high school or not. (A), (B), and (D) are all purposes of transition assessments and are, therefore, incorrect responses.Topic: Competency 005

A special education co-teacher is working with a fifth-grade student with learning disabilities. As the student works through a series of math problems, the teacher observes that word problems, particularly ones with multiple steps, seem particularly challenging for the student. The student often does just the first operation he can figure out and then abandons the problem. Based on this informal assessment, what type of cognitive deficits is the student most likely exhibiting?

A. Visual-spatial deficits B. Memory deficits C. Procedural learning deficits D. Number sense deficits (C) The correct response is (C). Students with procedural learning deficits struggle to apply adequate sequencing when attempting to solve multistep computations or word problems. They are often unsure of which operation to use first or how to structure a problem. It is common for students with procedural learning deficits to use immature strategies like this student does when he completes the first operation and then abandons any further steps. (A) is incorrect because visual-spatial deficits tend to cause issues with geometry as well as number alignment, measurement, and place value. (B) is incorrect because memory deficits tend to result in trouble with math facts and fluency. (D) is incorrect because number sense refers to the most foundational concepts of math such as one-to-one correspondence.

Which of the following forms of communication would be considered two-way communication?

A. Weekly school newsletters B. Email blasts from the school principal C. Parent-teacher phone call D. Report card (C) The correct response is (C). A parent-teacher phone call is an example of two-way communication because both the parent and the teacher have an opportunity to speak and to listen. (A), (B), and (D) are all incorrect because newsletters, email blasts, and report cards are all examples of one-way communication.Topic: Competency 012

According to IDEA 2004 and CFR 300.43, transition services must be based on

A. a school's resources.B. local community opportunities for students with disabilities.C. parents' expectations for what their child can accomplish.D. an individual student's needs, taking into account strengths, preferences, and interests. (D) The correct response is (D). IDEA 2004 and its associated law, CFR 300.43, specifically state that transition services must be based on an individual student's needs and should take into account the student's strengths, preferences, and interests. (A) and (B) are incorrect because, while the availability of school and community resources will impact the transition goals, they should not be the basis for the plan. (C) is incorrect because transition is focused on the student's own desires as much as those can be communicated. The process of transition is ultimately about self-advocacy and the move to adulthood. Parents are part of the transition planning process, but their expectations are not the basis for the plan.

An elementary school hosts an open house a few days before school starts so that families can meet the teachers. Parents of a new student with autism come to the open house out of breath and scowling. They look around the classroom, and the mother immediately begins asking about types of sensory equipment she would like to see in the classroom. The father interrupts and says the student needs to be treated like every other student with no special equipment. The best way for the teacher to approach these parents is to

A. assume they are going to be difficult and maintain a professional but distant approach.B. approach both parents with a warm smile and excitement about the goals for the upcoming school year, and encourage the student to explore the classroom while asking the parents to talk about their son's strengths.C. address the mother and promise to ask the principal to purchase the sensory equipment.D. address the father and assure him that the son will have access to the standard state curriculum. (B) The correct response is (B). Working with families of children with disabilities requires empathy and tact. Many families experience conflict as parents struggle to understand and accept their child's learning difficulties. Teachers play a critical role in helping families access information, listening to families' concerns, and keeping the focus on the child and what is best for her or him. In this example, the teacher should maintain a friendly demeanor and an excitement about the opportunity to work with the child. Refocusing the parents on discussing the child's strengths will provide the teacher with valuable information and turn the event into a more positive outing for everyone. (A) is incorrect because assuming the parents will be difficult to deal with leads to an adversarial, defensive relationship from the beginning. (C) and (D) are both incorrect because favoring one parent over another will only lead to further conflict.

A middle-school teacher is working with a small group of below-level readers. He brings in five print advertisements for various consumer products and uses sticky notes to cover all the words. He then displays the advertisements one at a time and asks the students to tell him what each advertisement is trying to tell consumers. The purpose of this activity is to

A. convince the students to buy the products.B. show students how visual images in media communicate a message and how to interpret them.C. motivate reluctant readers by removing all the words.D. help students visualize while reading. (B) The correct response is (B). Messages, particularly in our media-rich world, are conveyed with more than just words. Helping students understand this concept through advertisements that have had their words removed or hidden helps students become more critical readers and viewers. It also helps struggling readers learn that they can communicate with more than just words. (A) is incorrect because classroom discussions are never about trying to "sell" a product. (C) is incorrect because the purpose for removing the words is to focus attention on how the visuals alone tell a story. (D) is incorrect because strategies to help students visualize generally involve students drawing their own pictures of what they see when they read.

The reason that Section 504 of the Rehabilitation Act was ignored by state and local educational agencies for 20 years after it was passed in 1973 is that the law

A. didn't make sense to school boards. B. was discriminatory .C. was never signed by the President of the United States. D. did not include provisions for federal funding or monitoring. (D) The correct response is (D). Section 504 of the Rehabilitation Act was ignored for 20 years by state and local educational agencies because it did not include provisions for federal funding or monitoring. (A) is incorrect because the law did make sense, but it involved expenditures school boards did not want to make. (B) is incorrect because the law was not discriminatory. In fact, it was clearly a civil rights legislation. (C) is incorrect because it was signed into law; it just was an unfunded mandate.

A fourth-grade teacher has a class of 22 students, including four students with IEPs and two English learners. At the beginning of the school year, the teacher administers an inventory with questions about how students approach different learning tasks and what subjects they like best. She also structures a variety of activities, including a memory game similar to Concentration, a classroom scavenger hunt for clues to the answer to a question, and a math game with manipulatives. As students complete the activities, the teacher observes and makes notes on a clipboard. The teacher is most likely

A. figuring out who the smartest students are.B. observing and noting each student's learning strengths so that she can plan appropriate activities.C. grouping students by ability.D. making sure every student likes her. (B) The correct response is (B). The teacher is providing learning activities that cover all of the major modalities: visual, auditory, tactile, and kinesthetic. Between her observations during the activities and the students' answers to the informal inventory, she will have a good grasp of each student's learning strengths so she can plan instruction accordingly. (A) is not the correct answer because the teacher's purpose is to assess all learners, and her classroom clearly reflects her grasp of Gardner's theory of multiple intelligences. (C) is not the correct answer because she is clearly not grouping by ability. Rather, she is looking at which learners need more visual supports versus which ones need more tactile/kinesthetic activities. (D) is not the correct answer because the informal assessment is not about getting students to like her but instead about understanding her students and their learning strengths.

When planning instruction for students with special needs, the best approach is to

A. focus on students' weaknesses. B. follow the lessons in textbooks and programs exactly. C. teach to students' strengths. D. teach all students the same way. (C) The correct response is (C). Teaching to a student's strengths will ensure both more rapid success and increased self-esteem. (A) is incorrect because while a teacher may target the skills gaps a student has, focusing solely on weaknesses can be overwhelming and disheartening for a student. (B) is incorrect because part of the role of a special education teacher is to make appropriate adaptations to textbooks and programs to meet an individual student's needs. (D) is incorrect because the key to special education is not only recognizing but planning for the differences in students' learning patterns.

A sixth-grade mathematics teacher uses her interactive whiteboard and individual student clickers to administer informal quizzes during her lessons at least twice a week. The most appropriate reason for this teacher to give students regular informal quizzes is to

A. get enough individual grades for an accurate average. B. determine whether a student should be recommended for special education testing. C. utilize the expensive classroom technology. D. identify skills that the teacher should reteach. (D) The correct response is (D). Regular informal quizzes are examples of formative assessment. This assessment is best used to identify skills that students have not yet mastered and that the teacher, therefore, needs to reteach, preferably in a different way. (A) is incorrect as the reason behind quizzes should not be simply to get grades. Quizzes can be excellent tools for using assessment to inform instruction. (B) is incorrect because performance on quizzes should not be the sole qualifier for special education testing. (C) is incorrect because an interactive whiteboard, like any classroom tool, exists to help teachers reach their students, not to be used simply for the sake of technology.

One week before a geography teacher begins the next unit in the textbook, her co-teacher conducts an activity where he provides each student with half an index card with part of a word on it. Students have to stand and find the student who has the other half of their card. Then they work with their partner to find the definition for the vocabulary word on their card and draw a picture of what the word represents. The purpose of this lesson is to

A. give students a break from the textbook. B. preteach vocabulary and thus improve students' ability to read and understand the next chapter. C. show that all geography-related vocabulary words make great pictures. D. give the co-teacher a chance to conduct a lesson. (B) The correct response is (B). Preteaching vocabulary is a research-based strategy for improving reading comprehension. In a content-area class such as geography, students encounter a number of academic vocabulary words that they must understand to grasp central concepts. A hands-on vocabulary strategy like the one described helps students take ownership of the words they will encounter in the unit. (A) is incorrect because, while the activity is a break from the textbook, that is not the reason for the activity. (C) is incorrect because not all geography terms are easily visualized. (D) is incorrect because part of co-teaching is conducting lessons when appropriate and coaching from the side at other times. It all depends on the dynamic between the co-teacher and the general education teacher.

When talking to parents on the phone or at a parent-teacher conference, the most important thing a teacher can do is to

A. have documented examples for every problem the teacher has with the student.B. have an answer for every question parents ask.C. praise the parents for their outstanding parenting skills.D. take time to listen to the parents' responses and concerns, and note anything that requires follow-up. (D) The correct response is (D). When talking with parents, the most important thing teachers can do is be respectful and responsive listeners. Parents have a wealth of information about their child and can often help problem-solve situations or anticipate where conflicts might occur. In any conversation, when participants feels they have an opportunity to speak and truly be listened to, they tend to feel more positive about the situation and more willing to compromise to find a solution. (A) is incorrect because, while documentation and examples are good things to have, going into a phone call or conference with only problems is likely to lead to negative interactions. (B) is incorrect because teachers do not need to have every answer in a conversation. If a teacher does not have an answer, the best approach is to be honest and then note the question and promise to follow up and get the answer. (C) is incorrect because praise, when genuine, is a positive aspect to the conversation, but it is not the most important element of a problem-solving parent-teacher conference.

The primary reason to provide assessment accommodations is to

A. help students with disabilities have an equal opportunity to demonstrate what they know. B. give an advantage to students with disabilities .C. make sure students with disabilities pass their classes. D. make the test administrator's job more challenging. (A) The correct response is (A). Assessment accommodations do not give students with disabilities an advantage over other students taking the test. Instead, accommodations "level the playing field" so that students have an equal opportunity to demonstrate what they know. (B) and (C) are simply not true. (D) is incorrect because the purpose of accommodations has nothing to do with the test administrator.

The purpose of a transition plan in a student's IEP is to

A. help the student move from elementary to middle school. B. help the student move from middle school into high school. C. help the student move from high school to postgraduation and adult life. D. help students move from special education to general education. (C) The correct response is (C). By IDEA definition, the purpose of a transition plan is to help a student successfully move from high school to postgraduation and adult life. While (A), (B), and (D) all represent times of transition in a student's life, they do not represent the period of time for which IDEA requires the IEP team to document a plan.

The purpose of a standardized achievement test is to

A. identify the best reading method to use with a student.B. measure the amount of knowledge and skills a student has acquired in comparison to a larger group.C. establish a baseline for learning prior to starting a new unit.D. help a teacher form instructional groups. (B) The correct response is (B). Standardized achievement tests score students in comparison to a larger group. These tests have gone through a series of pilot tests to ensure they are as statistically fair as possible. (A), (C), and (D) are all incorrect as these are not the purpose of standardized achievement tests.

Use the information below to answer questions 28 and 29.An elementary ALE teacher uses puff paint and index cards to create flash cards with raised words. The words on the cards are: and, is, what, I, am, to, have, see, the, you. The teacher gives each student several minutes to trace the raised words with their fingers. If the student is verbal, the teacher has the student say the word while looking at it and tracing it. If the student is nonverbal, the teacher says the word while the student looks at the word and traces it.The most likely reason that the teacher wants the word spoken at the same time the word is being traced is to

A. increase retention through repetition and the combination of learning modalities.B. make the activity fun.C. make sure the words are pronounced correctly.D. decrease disruptive behaviors. (A) The correct response is (A). By having students simultaneously look at the words, trace the raised letters, and say the word, the teacher is incorporating repetition, specifically repetition through different learning modalities: visual, tactile/kinesthetic, and auditory. This combination of repetition and multisensory learning is a powerful tool for building short- and long-term retention. (B) is not correct because the purpose is not fun, although students do enjoy completing the activity as they learn the sight words. (C) is not correct because the purpose is to have a multisensory approach to improve retention, but as students read the word, the teacher will model correct pronunciation. (D) is not correct because the purpose is academic, not behavioral. A side benefit of hands-on learning, however, is the decrease in disruptive behavior that often accompanies it.

One advantage of implementing a research-based Response to Intervention (RTI) program from a special education teacher's perspective is that RTI

A. is the regular education teacher's responsibility, thus leaving the special education department with more time. B. involves less work than special education diagnostic testing. C. is easy to implement. D. can potentially reduce overidentification of students with learning disabilities. (D) The correct response is (D). RTI provides increasingly intensive intervention as soon as learning deficits are demonstrated without the legal requirements for testing and eligibility that are part of special education. This early identification and intervention can correct learning deficits quickly and thus reduce the number of students who will later need special education services. (A) is incorrect because RTI is shared between regular education and special education. Neither one should be fully responsible for implementing an RTI program. (B) and (C) are incorrect because a quality RTI program involves a great deal of planning and teacher support.

Parents of a fifth grader with learning disabilities have a parent-teacher conference with the student's classroom teacher and special education co-teacher. The parents indicate that they would like to learn more about learning disabilities and the potential schools and camps that may be appropriate for their child. An appropriate resource for the teachers to share with the parents would be information about

A. local camps for children with autism.B. local group homes and other residential facilities.C. the location of local libraries.D. LDA and a link to the organization's website. (D) The correct response is (D). LDA is the Learning Disabilities Association of America. It is an organization of parents and teachers of children with disabilities. Many of its members are individuals with learning disabilities who advocate for themselves. The organization's website is www.ldanatl.org. Providing parents with this information and the link will give them information about learning disabilities and about a wide variety of schools and camps dedicated to helping individuals with learning disabilities. (A) is incorrect because camps for children with autism will not meet the needs of students with learning disabilities. (B) is incorrect because group homes and other residential facilities are placements for adults with intellectual disabilities. They are not appropriate placements for adults with learning disabilities. (C) is incorrect because it does not help the parents narrow their search for information.

Mrs. Seville, a seventh-grade mathematics teacher, creates an end-of-unit test for her geometry unit. All of the questions are open-ended and require students to show their work and write their answers. A special education co-teacher might modify this test for students with learning disabilities in Mrs. Seville's class by

A. making no modifications; all students must take the same test to be fair.B. converting most of the questions to multiple choice, but still require students to show their work on the paper.C. reading the test questions out loud for students.D. providing a calculator. (B) The correct response is (B). Students with learning disabilities, particularly language-learning disabilities, will be at a disadvantage with an entirely open-ended, short-answer assessment. Modifying the questions by making them multiple choice is an appropriate modification. The students are still expected to do the math and show their work; they are just not tasked with composing complete sentences. (A) is incorrect because one of the primary tasks of a special education teacher is to make appropriate accommodations and modifications to ensure a "level playing field." Making no modifications would be unfair to the students with learning disabilities. (C) and (D) are incorrect because they are accommodations (changes to the environment) and not modifications.

As part of the morning circle time in an elementary ALE classroom, the teacher has a small pocket chart with three pockets labeled "hundreds," "tens," and "ones." Each day, a student is selected to place a yellow straw in the "ones" pocket and then count all of the straws in that pocket. If there are 10 straws, the student gets to exchange them for a blue straw, which goes in the "tens" pocket. The mathematical concept the teacher is using to count the days of the school year is

A. number sense.B. place value.C. patterns.D. fractions. (B) The correct response is (B). Any time students are working with "hundreds," "tens," and "ones," they are focused on place value. This example also shows how a concept such as place value can be reinforced by integrating it into a regular routine like circle time. (A) is incorrect because number sense involves basic number concepts such as counting and one-to-one correspondence. (C) is incorrect because patterns focus on sequencing numbers or shapes using some kind of repeating rule. (D) is incorrect because fractions focus on parts of a whole like one-half or one-quarter.

A middle-school special education teacher is teaching a mathematics class to seventh graders who failed the sixth-grade STAAR math test. The teacher provides highlighters to each student and shows them a list of keywords to look for and highlight in a series of word problems. Each time the students are assigned word problems, the teacher has them highlight keywords. She tells her students that this technique will help them on the test. The strategy that the teacher is providing her students with is

A. pictorial representation. B. corrective feedback. C. generic problem-solving strategy. D. modeling strategy. (C) The correct response is (C). Teaching students to highlight keywords every time they encounter a word problem is a generic problem-solving strategy that can help struggling learners slow down and think about the clues the words provide for solving the problem. (A) is incorrect because a pictorial representation is either a graph or an actual drawing of pictures to help set up a problem. (B) is incorrect because corrective feedback is provided by a teacher as a student is completing a problem. Corrective feedback allows students to correct mistakes as they make them. (D) is incorrect because a modeling strategy involves the teacher or facilitator actually completing a problem step-by-step while students watch and listen

If a child cries easily, does not want to interact with peers, and does not like coming to school on a consistent basis, the best course of action to take would be to

A. report it to the school counselor and recommend behavioral observation by a school psychologist. B. ignore the behaviors, as all children sometimes have issues at school. C. seat the child with classmates who will be empathetic. D. suggest to the parents that they should talk to their child. (A) The correct response is (A). All of the listed symptoms (i.e., crying easily, avoiding interactions with peers, and not wanting to come to school) are potential symptoms of a behavioral or emotional disorder. Further observation and testing is called for in this case. (B) is incorrect as the frequency and severity of the behaviors indicate a child in an emotional crisis of some sort. (C) is incorrect because this student's symptoms indicate deeper issues than peers are capable of dealing with. (D) is incorrect because ethically a school employee is required to report behaviors like those listed to school authorities in the interest of the child.

A special education teacher has several students who speak Spanish at home, but the teacher is not bilingual. The best approach for this teacher to take when communicating with Spanish-speaking parents is to

A. send home written notes in English and instruct students to translate the notes for their parents.B. talk to parents using a lot of gestures to communicate meaning.C. arrange for a bilingual colleague to serve as an interpreter during all parent phone calls and parent-teacher conferences.D. send all communication via emails so the parents can use online translation programs. (C) The correct response is (C). When working with parents who do not speak English, it is imperative to find a translator at the school or via a translation phone service to ensure that teachers and parents can truly communicate. Making sure to have the translator at all meetings is a way for the teacher to show her respect for the parents and the language they speak. (A) is incorrect because many students, especially students with disabilities, cannot read English well enough to serve as translators for their parents. Notes should be translated and sent home in the parents' native language if at all possible. (B) is incorrect because using a lot of gestures to communicate can be extremely insulting to parents and can lead to major miscommunication. (D) is incorrect because, just like notes, emails should be translated and sent in the parents' native language.Topic: Competency 012

The best way for dealing with a student who acts silly and talks loudly throughout class time is to

A. send the student to time out. B. ignore the behavior. C. create a leadership role for this student. D. make sure all the other students in the class see you reprimanding the student. (C) The correct response is (C). Acting silly and talking loudly are attention-seeking behaviors. This is a student who wants to be noticed. By assigning a leadership role in the class to this student, the teacher is providing an appropriate way for the student to be recognized. Neither (A) nor (B) are the best solutions because they are likely to escalate the attention-seeking behavior as they do not help the student achieve his ultimate goal to be noticed. (D) is not the best solution because it rewards the student's inappropriate behavior by providing him with negative attention.

A sixth-grade social studies teacher is having difficulty motivating several students who are tactile/kinesthetic learners. The best way to deal with the problem is to

A. show a series of images related to the upcoming unit on the interactive whiteboard.B. have the students read the next chapter in class, with each student expected to read a paragraph out loud.C. play a recording of sounds associated with the content of the next chapter and have students guess what they are.D. place sheets of chart paper around the room. On each one, write an open-ended question about a debatable issue related to the content of the next unit. Have students walk around the room and write their stance on each issue. (D) The correct response is (D). Tactile/kinesthetic learners retain information best when they are learning while moving. For example, a study strategy for a tactile/kinesthetic learner might be running laps while listening to recorded notes. Allowing students to move about the classroom while answering high-interest questions is a great way to motivate tactile/kinesthetic learners. (A) is incorrect because it describes a strategy for visual learners. (B) is incorrect because it describes the round-robin reading strategy, one that tends to demotivate struggling readers because of public embarrassment when they stumble or read too haltingly. (C) is incorrect because it describes a strategy for auditory learners.

Mrs. Benson, a third-grade teacher, has a diverse class that includes students with learning disabilities, English learners, and gifted students. For a reading unit, she pairs the students based on personality (for example, patient fluent readers with struggling readers). She then has the partners complete together every reading assignment in that unit. For text reading, partners decide how they will divide up the text for reading (for example, switching off by paragraphs or pages). For comprehension activities, partners turn in one paper representing work from both students. While partners are working together, Mrs. Benson walks around the classroom observing pairs and assisting with questions. One of the primary advantages of Mrs. Benson's paired reading strategy is that

A. struggling students can let their partners do all their work. B. students get a great deal more supported oral reading practice than they would in a group-reading situation. C. Mrs. Benson can take a break from lesson planning. D. students are not paired with their friends so there is less off-task talking. (B) The correct response is (B). Paired reading is an excellent method of providing peer support for reading assignments while freeing the teacher to monitor and work with pairs as needed. In a small-group setting, a teacher has to split instructional time as evenly as possible. In paired reading, every student has a partner to read to, and this facilitates more supported oral reading practice than students would receive in a group setting. (A) is incorrect because the teacher is monitoring the partners to ensure that both partners participate in the reading and the postreading assignment. (C) is incorrect because lessons requiring that all the students be engaged in a group task actually take deeper planning to help prevent classroom management issues. (D) is incorrect because pair selection is not based on friendship status; it is based on reading ability and complementary personalities.

Mr. Osborn, an ALE teacher, works one-on-one with each of his students. Early in the school year, he presents each student with a list of words. He says, "I am going to show you a list of words you may or may not know. If you know the word, read it out loud. If you do not know the word, say 'pass.' " As students read the list of words, Mr. Osborn makes notes on a separate checklist. He places a check mark next to any words the student reads correctly in a half second or less. He places an "X" next to any word the student cannot read, hesitates to read, or has to work to sound out. Mr. Osborn is most likely assessing

A. students' sight-word automaticity.B. students' grasp of the English language.C. students' ability to follow directions.D. students' reading comprehension. (A) The correct response is (A). Sight-word automaticity is a critical aspect of fluent reading. There are 200 frequently used words that make up over 50 percent of printed text in the English language. Many of these words do not follow typical phonetic conventions. For this reason, educators work to help students recognize these words on sight—thus the term "sight words." Automaticity is achieved when a student can call a word correctly in a half second or less. (B) is incorrect as the English language has many more aspects and nuances than a list of words could represent. (C) is incorrect as the test is focused on how well and how quickly students read the words. The directions will be repeated if students do not understand them. (D) is incorrect because this assessment is focused on fluency, not comprehension.

A special education teacher is working with a small group of students with documented social-skills goals in their IEPs. An appropriate activity for the small group to do would be

A. sustained silent reading.B. taking notes as the teacher talks about the benefits of good social skills.C. playing a board game built around conversation skills.D. writing goals about how many friends each student wants to make in the next month. (C) The correct response is (C). Improving social skills involves setting up scenarios where students can practice interacting with others. A board game focused on conversation skills provides an activity that encourages students to talk to each other and use questions and comments to carry on a conversation. (A) is incorrect because it is an independent activity with no social-skills practice. (B) is incorrect because taking notes and listening to a teacher do not build the student's own social skills. (D) is incorrect because social-skills instruction should focus on the actual practice of the skills that will help students make friends. There should not be an emphasis on how many friends to make, but on how to be at ease striking up a conversation and interacting with others.

Marcus is a ninth grader with intellectual disabilities. His transition needs evaluation indicates that he likes pets, especially dogs. He does not like doctors and becomes nervous around needles. Marcus is good at carrying out routine tasks. Based on this information, an appropriate supported employment opportunity for Marcus would be

A. taking animals at a veterinarian's office for walks. B. dealing with customer issues at a customer service counter. C. completing janitorial services at a local hospital .D. feeding and washing dogs at an animal shelter. (D) The correct response is (D). Based on Marcus's interest in pets, particularly dogs, and his aptitude for routine tasks, a supported employment opportunity feeding and washing dogs at an animal shelter would be an appropriate choice. (A) is not an appropriate choice because, while there are pets at a veterinarian's office, there are also needles and medical equipment, which Marcus does not like. (B) is not an appropriate choice because dealing with customer issues requires split-second decision making and constantly changing expectations. Marcus needs a job that is predictable where he can learn the routines. (C) is not an appropriate choice because a hospital will have doctors and needles, both things that make Marcus uncomfortable.Topic: Competency 007

In special education professional circles, the term "disproportionality" refers to

A. the overrepresentation of racial, cultural, and linguistic minorities in special education. B. a mathematical dilemma with geometric shapes. C. the high number of students with autism in ALE classrooms. D. the low number of Texas schools receiving special education funding. (A) The correct response is (A). The term "disproportionality" in special education refers to the overrepresentation of racial, cultural, and linguistic minorities in special education. This is a critical issue in special education and one that special education teachers should constantly watch for in their own evaluations and assumptions. (B), (C), and (D) are all incorrect as they are not true.

A student with an IEP is caught defacing a school wall. Based on the school's written code of conduct, the assistant principal suspends the student from school for three days. The student has not been suspended previously that school year, but the parents protest the suspension, saying that the student's status as a special education student prevents him from being suspended. The most appropriate response is that

A. the school district should override the suspension since the student receives special education services.B. the principal should ask the assistant principal to tell the parents that if they pay for the damages, the student will not be suspended.C. the suspension should be changed to in-school suspension so that the parents do not have to arrange for child care.D. the assistant principal should call a meeting with the parents with another administrator present and point out that disciplinary sanctions of less than 10 days are not considered a change of placement under IDEA and are, therefore, legal. (D) The correct response is (D). Under IDEA, disciplinary sanctions are not considered a change in placement and are not subject to this restriction provided the student has not been suspended for more than 10 days cumulatively during the school year. To help ensure a good relationship between the school and parents, the assistant principal should share this information with the parents in a nonconfrontational context with a witness. (A) is incorrect because IDEA makes clear that receiving special education services does not prevent a school from implementing disciplinary sanctions, particularly if the misbehavior is not related to the disability. (B) is incorrect because such a proposal would be similar to bribing the parents in order to avoid the suspension. Reparation of damages can be part of the disciplinary sanction, but it is not a replacement for the assistant principal's original decision. (C) is incorrect because the parents' need for childcare is not a factor in the implementation of the disciplinary process.

Look at the following test results. Standardized Test Results Name Grade Adjusted Score Percentile Score Jane Doe 5 215 65 The percentile score means that

A. the student passed the test. B. the student failed the test. C. 65 percent of students who took the test scored equal to or less than the test-taker. D. 35 percent of students who took the test scored less than the test-taker. (C) The correct response is (C). A percentile rank indicates the percentage of a reference or norm group obtaining scores equal to or less than the test-taker's score shown in the table. (A) and (B) are incorrect because the purpose of a percentile rank is not to indicate a passing or failing score. (D) is incorrect because a 65 percent ranking would indicate that 35 percent of students who took the test scored better than the test-taker.

A middle-school special education teacher reviews a student's grades as part of planning her IEP goals for the next ARD. She sees the following: Jan Doe Report Card Reading 80 Language Arts 60 Mathematics 88 Science 80 Social Studies 65 Physical Education 95 Music 84 When the teacher reflects on the general education teachers the student has, she knows that the reading teacher incorporates a lot of movement and critical thinking activities into her lesson plans. The language arts teacher requires students to write a paper every week as homework. The social studies teacher assigns chapters and gives open-ended unit tests. The science teacher has students conduct at least two experiments per week. The teacher might conclude that

A. the student wants to be an astronaut. B. the student only likes PE. C. the student may be a tactile/kinesthetic learner and needs some explicit reading and writing strategy instruction .D. the student may be an auditory learner and clearly does not like her social studies and language arts teachers. (C) The correct response is (C). The student's best grade is in PE, and her second-best grade is in a class that emphasizes hands-on experiences; so there are a couple of indications that she may be a tactile/kinesthetic learner. The special education teacher would have to confirm this through observation and learning styles inventories. The low grades in the content areas of language arts and social studies are a tip-off to the student's struggles with reading and writing. The fact that the reading teacher incorporates movement into her lesson plans may be part of the reason the reading grade is passing. (A) is incorrect as the student's good grade in science is not indicative of a career choice. (B) is incorrect because a high grade in any class is not definite evidence that a student likes that class exclusively. (D) is incorrect because there is no evidence of the student being an auditory learner, and poor grades do not always translate into dislike.

A special education teacher works with a biology teacher to create a mind map of the upcoming unit on plants. The most likely purpose for this collaboration is

A. to help both teachers identify the big ideas and how they are related to each other so that they can identify the key elements to assess as the unit progresses. B. for the biology teacher to be able to teach the special education teacher science concepts. C. to show that minds have branches and roots just like plants. D. for the special education teacher to show the biology teacher a cool new technique she just learned at a staff development meeting. (A) The correct response is (A). A mind map is a diagram that visually outlines key information. It is an excellent strategy for working through the key ideas of a unit of study. By collaborating on a mind map, the two teachers are figuring out the key concepts they will emphasize in instruction so that they can accurately assess student understanding during and after the unit. (B) is incorrect as this is collaborative planning, not one teaching the other. (C) is incorrect as the decision to use a mind map was not due to the topic being about plants. (D) is incorrect as the two teachers are using a common tool to plan together, not showing off a new idea.


Related study sets

Global History: Chapter 23-French Revoltion

View Set

Principles & Practices: Module 7 (Chapter 11)

View Set